+7 495 120-13-73 | 8 800 500-97-74

(для регионов бесплатно)

Содержание

Электроёмкость плоского конденсатора | Физика. Закон, формула, лекция, шпаргалка, шпора, доклад, ГДЗ, решебник, конспект, кратко

Плоским конденсатором обычно называ­ют систему плоских проводящих пластин — обкладок, разделенных диэлектриком. Про­стота конструкции такого конденсатора по­зволяет сравнительно просто рассчитывать его электроемкость и получать значения, совпадающие с результатами эксперимента.

Рис. 4.71. Плоский конденсатор на элект­рометре

Укрепим две металлические пластины на изоляционных подставках и соединим с электрометром так, что одна из пластин будет присоединена к стержню электромет­ра, а вторая — к его металлическому кор­пусу (рис. 4.71). При таком соединении электрометр будет измерять разность по­тенциалов между пластинами, которые об­разуют плоский конденсатор из двух пла­стин. Проводя исследования, необходимо пом­нить, что

при постоянном значении заряда пластин уменьшение разности потенциалов свидетельствует об увеличении электроем­кости конденсатора, и наоборот.

Сообщим пластинам разноименные заря­ды и отметим отклонение стрелки электро­метра. Приближая пластины друг к другу (уменьшая расстояние между ними), заме­тим уменьшение разности потенциалов. Та­ким образом, при уменьшении расстояния между пластинами конденсатора его элект­роемкость увеличивается. При увеличении расстояния показания стрелки электрометра увеличиваются, что является свидетельст­вом уменьшения электроемкости.

Электроемкость плоского конденсатора об­ратно пропорциональна расстоянию между его обкладками.

C ~ 1 / d,

где d — расстояние между обкладками.

Рис. 4.72. График зависимости емкости плоского конденсатора от расстояния между пластинами
Рис. 4.73. При расчетах емкости плоских конденсаторов учитывают площадь пере­крытия пластин

Эту зависимость можно изобразить гра­фиком обратной пропорциональной зависи­мости (рис. 4.72).

Будем смещать пластины одну относи­тельно другой в параллельных плоскостях, не изменяя расстояния между ними.

При этом площадь перекрытия пластин будет уменьшаться (рис. 4.73). Увеличение разности потенциалов, отмеченное электрометром, будет свидетельствовать об умень­шении электроемкости.

Увеличение площади перекрытия пластан приведет к увеличению емкости.

Электроемкость плоского конденсатора про­порциональна площади пластин, которые пере­крываются.

C ~ S,

где S — площадь пластин.

Рис. 4.74. График зависимости емкости плоского конденсатора от площади его пластин

Эту зависимость можно представить гра­фиком прямой пропорциональной зависи­мости (рис. 4.74). 

Возвратив пластины в начальное поло­жение, внесем в пространство между ними плоский диэлектрик. Электрометр отметит уменьшение разности потенциалов между пластинами, что свидетельствует об увели­чении электроемкости конденсатора. Если между пластинами поместить другой диэлек­трик, то изменение электроемкости будет иным.

Электроемкость плоского конденсатора за­висит от диэлектрической проницаемости ди­электрика.

C ~ ε,

где ε — диэлектрическая проницаемость ди­электрика. Материал с сайта http://worldofschool.ru

Рис. 4.75. График зависимости емкости плоского конденсатора от диэлектри­ческой проницаемости диэлектрика

Такая зависимость показана на графике рис. 4.75.

Результаты опытов можно обобщить в ви­де

формулы ёмкости плоского конденсатора:

C = εε0S / d,

где S — площадь пластины; d — расстояние между ними; ε — диэлектрическая прони­цаемость диэлектрика; ε0 — электрическая постоянная.

Конденсаторы, которые состоят из двух пластин, в практике применяются очень редко. Как правило, конденсаторы имеют много пластин, соединенных между собой по определенной схеме.

На этой странице материал по темам:
  • Решение задач по теме электроемкость плоского конденсатора

  • Заключение по электроемкости

  • Теория плоских конденсаторов

  • График зависимости емкости c плоского конденсатора

  • Простые задачи энергия плоского конденсатора скачать

Вопросы по этому материалу:
  • Какое строение плоского конденсатора?

  • По изменению какой величины в опыте можно делать заключение об изменении электроемкости?

  • В какой последовательности проводится опыт, в котором устанавли­валась зависимость электроемкости конденсатора от его параметров?

  • Как зависит электроемкость плоского конденсатора от активной площади пластин?

  • Как зависит электроемкость плоского конденсатора от расстояния между пластинами?

  • Как влияет диэлектрик на электроемкость конденсатора?

Вычисление емкости конденсатора.

Наука техника технологии

Плоский конденсатор – это физическое упрощение, взявшее начало из ранних исследований электричества, представляющее собой конструкцию, где обкладки имеют форму плоскостей и в каждой точке параллельны.

Формулы

Многие ищут формулы, описывающие ёмкость плоского конденсатора. Если это так, то не читайте ниже любопытные и малоизвестные факты, потому что сухие математические знаки, конечно же, важнее.

Первым определил ёмкость плоского конденсатора Вольта. В его распоряжении ещё не было такой величины, как разница потенциалов, именуемая напряжением, но интуитивно он совершенно правильно объяснил суть явления. Что касается количества зарядов, то он трактовал её, как объем электрического флюида атмосферы – не совсем правильно, но в конечном итоге похоже на правду. Согласно этому мировоззрению ёмкость плоского конденсатора может быть найдена, как отношение объёма накопленного электрического флюида к разнице атмосферных потенциалов, то есть:

Эта формула применима к любому конденсатору, вне зависимости от его конструкции. То есть, является универсальной. Специально для плоских конденсаторов имеется формула ёмкости, выраженная через свойства материала диэлектрика и геометрические размеры:

В этой формуле через S обозначена площадь обкладок, вычисляемая через произведение сторон, а d – показывает расстояние между обкладками. Прочие символы – электрическая постоянная (8,854 пФ/м) и диэлектрическая проницаемость материала диэлектрика, да простит Тот столь откровенные тавтологии. Электролитические конденсаторы обладают столь большой ёмкостью по той причине, что проводящий раствор отделен от металла очень тонким слоем оксида. Следовательно, d в этом случает будет минимальным. Единственный минус в том, что электролитические конденсаторы полярные, их нельзя подключать в цепи переменного тока. С этой целью на каждом анод или катод обозначены значками плюса или минуса.

Плоские конденсаторы сегодня редко встречаются, и это преимущественно плёночные микроскопические технологии, где такой род поверхностей является доминирующим. Все пассивные и активные элементы образуются через трафарет. И, следовательно, имеют вид плёнок. Плоские индуктивности, резисторы и конденсаторы наносятся в виде токопроводящих паст.

От материала диэлектрика ёмкость зависит по той причине, что у каждого из них структура своя. Считается, что аморфное вещество состоит из неориентированных диполей, упруго укреплённых на своих местах. При приложении внешнего электрического поля они обратимо ориентируются вдоль силовых линий, ослабляя напряжённость. В результате заряд продолжает накапливаться, пока этот процесс не прекратится. По мере выхода энергии из обкладок диполи возвращаются на свои места, делая возможным следующий рабочий цикл. Так функционирует плоский электрический конденсатор.


Из истории

Исторически первым начал исследовать накопление заряда великий Алессандро Вольта. В докладе Королевскому научному обществу за 1782 год он впервые озвучил слово конденсатор. В понимании Вольты электрофорус, представляющий собой две параллельные обкладки, выкачивал из эфира электрический флюид.

В то время все познания сводились к тому, что учёные думали, будто атмосфера Земли содержит в себе нечто, что не может быть определено приборами. Существовали только простейшие электроскопы, способные определить знак заряда и его наличие, но не дававшие представления о количестве. Учёные просто натирали мехом поверхность тела и подносили его для исследования в область влияния прибора. Ещё Гильберт показал, что электрические и магнитные взаимодействия ослабевают с расстоянием. Поэтому учёные примерно знали, что нужно делать, но исследования не продвигались ни на йоту.

Гипотеза об атмосферном электричестве высказана Бенджамином Франклином. Он активно исследовал молнии и пришёл к выводу, что это проявления все той же единой силы. Запуская воздушного змея в небо, он соединял его шёлковой нитью с землёй и наблюдал дуговой разряд. Это были достаточно опасные опыты, и Бенджамин много раз рисковал своей жизнью ради развития науки. О том, что шёлковая нить проводит статический заряд, было известно от Стивена Грея, первым собравшего в 1732 году электрическую цепь.

Уже через 20 лет (1752 год) Бенджамин Франклин предложил конструкцию первого громоотвода, осуществлявшего молниезащиты близлежащих построек. Только вдуматься! – до этого всякий мог ожидать того, что его дом сгорит от случайного удара. Именно Бенджамин Франклин предложил один из видов заряда называть положительным (стеклянный), а другой отрицательным (смоляной). Так физики были введены в заблуждение относительно истинного направления движения электронов. Но как они могли думать иначе, когда в 1802 году на примере опытов нашего соотечественника Петрова увидели, что на аноде образуется ямка? Следовательно, положительные частицы переносили заряд на катод, вот только это были ионы воздушной плазмы.

К началу исследования Вольтой электрических явлений, таким образом, были уже известны статические заряды и факт наличия у них двух знаков, кроме того люди упорно считали, что весь «флюид» берётся из воздуха. На эту мысль их натолкнули опыты с натиранием янтаря шерстью, которые не могли быть проведены под водой. Следовательно, логичным было предположить, что электричество может происходить только из атмосферы Земли, что, конечно же, совершенно неверно. В частности, многие растворы, исследованные Хампфри Дэви, могут проводить электрический ток.

Причина, следовательно, была в другом – при натирании янтаря под водой силы трения снижались в десятки и сотни раз, а заряд рассеивался по всему объёму жидкости. Следовательно, этот процесс был всего лишь неэффективным. Но сегодня каждый добытчик знает, что нефть прекрасно электризуется трением о трубы и без воздуха. Следовательно, атмосфера для «флюида» не является обязательным компонентом.

Самый большой в мире плоский конденсатор

Столь систематизированные, но в корне неверные толкования все-таки не смогли остановить Вольту на его исследовательском пути. Он упорно изучал электрофорус, как один из самых совершенных генераторов, существовавших в то время. Вторым был серный шар Отто фон Герике, изобретённый более чем за век до этого (1663 год). С тех самых пор его конструкция мало менялась, но после открытий Стивена Грея заряд начали снимать при помощи проводников. В частности, в для этого служат металлические гребёнки-нейтрализаторы.

Долгое время учёные ходили вокруг да около. Электрофорная машина 1880 года может считаться первым мощным генератором разряда, позволявшим получить дугу, но своей настоящей силы электроны достигли в генераторе Ван де Граафа (1929 год), где разница потенциалов составила единицы мегавольта. Для сравнения грозовое облако, согласно данным Википедии, может иметь потенциал относительно Земли в единицы гигавольт (на три порядка больше, чем в человеческой машине).

Суммируя сказанное, можно с определённой долей уверенности сказать, что природные процессы используют в качестве принципа своего действия электризацию трением, влиянием и некоторые другие её виды, а мощный циклон является самым большим из известных нам плоских конденсаторов. Молния показывает, что бывает, когда диэлектрик (атмосфера) не выдерживает приложенной разницы потенциалов и пробивается. В точности то же самое происходит в любом плоском конденсаторе, созданном человеком, если вольтаж оказывается для него непомерным. Пробой твёрдого диэлектрика необратим, а возникающая электрическая дуга часто служит причиной расплавления обкладок и выхода изделия из строя.

Электрофорус

Итак, Вольта взялся за исследование модели природных процессов. Первый электрофорус появился в 1762 году сконструированный Йоханом Карлом Вильке. По-настоящему популярным прибор становится после докладов Вольты Королевскому научному обществу (середина 70-х годов XVIII века). Вольта же и дал прибору его нынешнее название.


Электрофорус способен накапливать электростатический заряд, образованный трением резины куском шерсти. Он состоит из двух плоских, параллельных друг другу обкладок:

  • Нижняя представляет собой тонкий кусок резины. Толщина его выбирается из соображений эффективности устройства. Если выбрать кусок более солидный, то значительная часть энергии будет накапливаться внутри диэлектрика на ориентацию его молекул. Что и наблюдается в современном плоском конденсаторе, куда диэлектрик помещается для увеличения электроёмкости.
  • Верхняя пластина из тонкой стали кладётся сверху, когда заряд уже накоплен трением. За счёт влияния на верхней поверхности образуется избыток отрицательного заряда, и он должен быть снят на заземлитель, чтобы при расстыковке двух обкладок не произошло взаимной компенсации.

Принцип действия этого плоского конденсатора должен быть уже понятен. Оператор трёт резину шерстью, оставляя на ней отрицательный заряд. Затем сверху кладётся кусок металла. Из-за значительной шероховатости поверхностей они не соприкасаются, но находятся на некотором расстоянии друг от друга. В результате металл электризуется влиянием. Электроны отталкиваются поверхностным зарядом резины и уходят на внешнюю плоскость, где оператор их снимает через заземлитель лёгким кратковременным прикосновением.

Низ металлической обкладки остаётся заряженным положительно. При расстыковке двух поверхностей этот эффект сохраняется, потому что в материале наблюдается дефицит электронов. И можно наблюдать искру, если дотронуться до металлической обкладки. Этот опыт можно на одном и том же заряде резины проделывать сотни раз, поскольку её поверхностное статическое сопротивление весьма велико. Это не даёт заряду растекаться. Демонстрируя этот опыт, Вольта привлёк внимание всего научного мира, но исследования никак не двигались вперёд, если не считать открытий Шарля Кулона.

В 1800 году сам Алессандро даёт толчок развитию изысканий в области электричества, изобретя свой знаменитый гальванический источник питания.

Конструкция плоского конденсатора

Электрофорус по сути представляет собой первый из когда-либо сконструированных плоских конденсаторов. Его обкладки способны хранить только статический заряд, потому что иначе наэлектризовать резину невозможно. Поверхность очень долго хранит электроны. Вольта даже предлагал снимать их пламенем свечи через ионизированный воздух или ультрафиолетовым излучением Солнца. Сегодня каждый школьник знает, что то же самое можно проделать и водой. Правда, электрофорус нужно будет после этого высушить.

В современном мире нижней обкладкой может служить тефлоновое покрытие или пластик. Они тоже хорошо набирают статический заряд. Диэлектриком здесь служит воздух. Чтобы перейти к конструкции современного конденсатора, нужно обе обкладки сделать металлическими. Тогда при возникновении на одной из них заряда влиянием электризация распространится на вторую, и если другой контакт заземлён, накопленная энергия может храниться какое-то время.


Запас электронов напрямую зависит от материала диэлектриков. Так например, среди современных конденсаторов встречаются:

  1. Слюдяные.
  2. Воздушные.
  3. Электролитические (оксидные).
  4. Керамические.

В эти названия как раз и заложен материал диэлектрика. От его состава зависит напрямую ёмкость, которая может быть увеличена во много раз. Роль диэлектриков объяснялась выше, в частности их параметры определяются непосредственно строением вещества. Однако многие материалы, обладающие высокими характеристиками, использовать не удаётся по причине их непригодности. Так например, вода обладает высокой диэлектрической проницаемостью.

Характеристика плоского конденсатора, мера его способности накапливать электрический заряд.

Так как поле сосредоточено внутри конденсатора, то линии напряженности начинаются на одной обкладке и кончаются на другой, поэтому свободные заряды, которые возникают на разных обкладках, равны по модулю и противоположны по знаку. Под емкостью конденсатора понимается физическая величина, равная отношению заряда Q, накопленного в конденсаторе, к разности потенциалов (φ1 — φ2) между его обкладками

При небольших размерах конденсатор отличается значительной емкостью, не зависящей от наличия вблизи него других зарядов или проводников. Обкладкам конденсатора сообщают одинаковые по модулю, но противоположные по знаку заряды, что способствует накоплению зарядов, так как разноименные заряды притягиваются и поэтому располагаются на внутренних поверхностях пластин.

Под зарядом конденсатора понимают заряд одной пластины.

Так же есть:

Энергия конденсатора:

Ёмкость цилиндрического конденсатора:

Емкость сферического конденсатора:

В формуле мы использовали:

Электрическая ёмкость (ёмкость конденсатора)

Относительная диэлектрическая проницаемость

Электрическая постоянная

Плоский конденсатор состоит из двух параллельных пластин, разделённых небольшим зазором шириной , заполненным однородным диэлектриком.

Нам известно, что поле между двумя разноимённо заряженными пластинами с одинаковой по величине поверхностной плотностью равно, где,S– площадь каждой пластины. Напряжение между обкладками:

Используя определение емкости конденсатора, получаем:

Отметим, что полученная формула является приближенной, так как выведена без учета искажения поля у краев пластин. Расчет по этой формуле дает завышенное значение ёмкости и тем точнее, чем меньше зазор по сравнению с линейными размерами пластин.

Ёмкость сферического конденсатора.

Сферический конденсатор представляет собой систему двух концентрических сфер с радиусами и. Электрическое поле между обкладками сферического конденсатора согласно теореме Гаусса определяется зарядом внутренней сферы. Напряжение между обкладками равно:

.

Для ёмкости сферического конденсатора получаем:

Это формула точная.

Если , полученная формула переходит в выражение для ёмкости плоского конденсатора.

Ёмкость цилиндрического конденсатора.

Цилиндрический конденсатор составляет систему двух коаксиальных цилиндров с радиусами и, длиной.

Рассуждая аналогично выводу ёмкости сферического конденсатора, получаем:

..

Полученная формула является приближенной и при малом зазоре переходит в формулу емкости плоского конденсатора.

Соединение конденсаторов.

В практике для получения необходимых значений емкости используют соединения конденсаторов: а) последовательное, б) параллельное, в) смешанное (см. рисунок).


Ёмкость последовательного соединения конденсаторов.

Заряды последовательно соединенных конденсаторов равны , а напряжение на батарее. Из определения емкости следует:

Если , то(ёмкость последовательного соединения меньше наименьшей ёмкости в последовательном соединении).

Для последовательно соединенных конденсаторов емкость вычисляется по формуле:

Ёмкость параллельного соединения конденсаторов.

Заряд батареи равен сумме зарядов:

а напряжение . По определению емкости получаем:

Для параллельно соединенных конденсаторов:.

В случае одинаковых конденсаторов: .

Оценить емкость батареи (см. рисунок) .

Используя свойство бесконечности можно представить цепь в виде соединения (см. рисунок).

Для расчета ёмкости батареи получаем:

Откуда: , так как, то.

Лекция 7.

Диэлектрики в электрическом поле.

Диэлектриками (изоляторами) называют вещества, не проводящие постоянного электрического тока. Это означает, что в диэлектриках отсутствуют «свободные» заряды, способные перемещаться на значительные расстояния.

Диэлектрики состоят либо из нейтральных молекул, либо из ионов, находящихся в узлах кристаллической решетки. Сами же молекулы могут быть полярными инеполярными. Полярные молекулы обладают дипольным моментом, у неполярных молекул дипольный момент равен нулю.

Поляризация.

В электрическом поле диэлектрики поляризуются. Это явление связано с появлением в объеме и на поверхности диэлектрика «связанных » зарядов. При этом конечный объем диэлектрика приобретает дипольный момент. Механизм поляризации связан с конкретным строением диэлектрика. Если диэлектрик состоит из неполярных молекул, то в пределах каждой молекулы происходит смещение зарядов – положительных по полю, отрицательных против поля, т.е. молекулы, приобретают дипольный момент. У диэлектрика с полярными молекулами в отсутствии внешнего электрического поля их дипольные моменты ориентированы хаотично.

Под действием электрического поля диполи ориентируются преимущественно в направлении поля. Рассмотрим подробнее этот механизм (см. рисунок). Пара сил исоздает вращательный момент равный, где- дипольный момент молекулы. Этот момент стремится ориентировать диполь вдоль поля. В ионных кристаллах под действием электрического поля все положительные ионы смещаются по полю, отрицательные – против поля. Отметим, что смещение зарядов очень малы даже по сравнению с размерами молекул. Это связано с тем, что напряженность внешнего электрического поля обычно много меньше напряженности внутренних электрических полей в молекулах.

Отметим, что существуют диэлектрики, поляризованные даже при отсутствии внешнего поля (электреты, сегнетоэлектрики). Мы остановимся на рассмотрении только однородных диэлектриков, в которых отсутствует остаточная поляризация, а объемный и «связанный» заряд всегда равен нулю .

КОНДЕНСАТОР — означает накопитель. В радио и электронной аппаратуре конденсатор является накопителем электрических зарядов. Простейший конденсатор состоит из двух металлических пластинок разделенных слоем диэлектрика. Диэлектрик — это материал который не проводит электрического тока и обладает определенными свойствами о которых поговорим чуть позже.

Так как конденсатор является накопителем, то он должен обладать определенной емкостью (объемом для накопления зарядов). На емкость конденсатора влияют площадь пластин (еще их называют «обкладками»), расстояние между обкладками и качество диэлектрика. К хорошим диэлектрикам относятся вакуум, эбонит, фарфор, слюда, полиэтилен, текстолит и много других синтетических материалов.
На рисунке изображен простейший конденсатор с двумя параллельными обкладками площадью S (S = m * n), которые находятся в вакууме на расстоянии d друг от друга.


Если между верхней и нижней обкладками конденсатора приложить напряжение Uab, то на верхней и нижней обкладках конденсатора накопятся одинаковые положительный +q и отрицательный -q заряды, которые называют свободными. Между обкладками возникает электрическое поле обозначенное на рисунке буквой Е.
Емкость нашего конденсатора (обозначается буквой С) будет: С = Eo*S/d, где Ео — электрическая постоянная (для вакуума) Ео=8,854 * 10 -12 Ф/м (Фарад на метр).
Если между обкладками поместить диэлектрик,


то ёмкость конденсатора будет: С = Er * Eo *S / d. В формуле расчета ёмкости добавилась величина Er — относительная диэлектрическая проницаемость введённого диэлектрика.
Из формулы следует, что емкость конденсатора увеличивается на величину Er проницаемости диэлектрика. Итак, чем больше площадь S пластин конденсатора, больше значение Er и меньше расстояние d между пластинами, тем больше емкость конденсатора. Основной единицей емкости в системе единиц СИ является фарад (Ф). Емкость 1Ф очень велика. В электротехнике обычно используют дольные единицы емкости:
микрофарада (мкФ), 1мкФ = 1*10 -6 Ф,
нанофарада (нФ), 1нФ = 1*10 -9 Ф, и
пикофарада (пФ), 1пФ = 1*10 -12 Ф.




При выборе диэлектрика для конденсаторов, кроме относительной диэлектрической проницаемости диэлектрика, учитывают еще два важных параметра:
1) Электрическую прочность — прочность диэлектрика при подаче на прокладки конденсатора высокого напряжения. При низкой электрической прочности может произойти электрический пробой, и диэлектрик станет проводником электрического тока;
2) Удельное объемное сопротивление — электрическое сопротивление диэлектрика постоянному току. Чем больше удельное сопротивление диэлектрика, тем меньше утечка накопленных зарядов в конденсаторе.

КОНДЕНСАТОР В ЦЕПИ ПОСТОЯННОГО ТОКА. На графике накопление заряда конденсатором выглядит как показано на рисунке 1.


Время заряда конденсатора зависит от ёмкости конденсатора (при одинаковом приложенном напряжении). Чем больше ёмкость конденсатора, тем больше время заряда. Аналогичная картина (Рис. 2) наблюдается при разрядке конденсатора на сопротивление. При одинаковом сопротивлении время разряда больше у конденсатора с большей ёмкостью.

КОНДЕНСАТОР В ЦЕПИ ПЕРЕМЕННОГО ТОКА. Если напряжение приложенное к емкостному элементу, будет изменяться по амплитуде (переменное напряжение),то будет изменяться и заряд конденсатора, то есть в емкостном элементе появится ток.



Ток Ic проходящий через конденсатор зависит от частоты f приложенного переменного напряжения и ёмкости С конденсатора. Если для постоянного тока сопротивление конденсатора можно считать равным бесконечности, то для переменного тока конденсатор обладает определённым сопротивлением. Сопротивление конденсатора переменному току Rc рассчитывается по формуле показанной на рисунке.
В формуле расчета емкостного сопротивления переменному току частота выражается в герцах, а емкость конденсатора в фарадах. Из формулы видно, что с увеличением частоты f при неизменной емкости конденсатора сопротивление Rc снижается, аналогично с увеличением емкости конденсатора при неизменной частоте сопротивление Rc так же снижается. Конденсаторы, так же как и резисторы, для получения заданной емкости Со можно включать параллельно и последовательно. Формулы расчета результирующей емкости показаны на рисунке.



КОНСТРУКЦИЯ, ПАРАМЕТРЫ И ТИПЫ КОНДЕНСАТОРОВ. Предположим, что мы конструируем конденсатор и попробуем, уже обладая определенными знаниями, рассчитать емкость конденсатора. Как известно, емкость конденсатора зависит от площади обкладок S, расстояния между обкладками d и диэлектрической проницаемости применяемого диэлектрика Er. Обкладки конденсатора изготавливаются из металлов с хорошей электрической проводимостью — алюминий, медь, серебро, золото. Емкость конденсатора не зависит от толщины обкладок, поэтому чем тоньше обкладки конденсатора, тем лучше — экономим металл и уменьшаем геометрический объём конденсатора.


Расстояние d не должно быть слишком малым, во избежание электрического пробоя диэлектрика.
Выберем в качестве диэлектрика наиболее распространенный материал — гетинакс с Er равной 6 … 8. Примем Er для нашего конденсатора равной 7.


Площадь S вычисляется для одной обкладки конденсатора при условии, что линейные размеры обкладок одинаковы. Если одна из обкладок имеет меньшие длину или ширину то площадь вычисляется для меньшей обкладки.
Все размеры — длина и ширина обкладок и расстояние между ними должны быть выражены в метрах. Примем размеры такие, какие показаны на рисунке. Подставим в формулу расчета емкости конденсатора наши данные: C = Er * Eo * S / d;
C = 7 * 8.854*10 -12 * 0.0025 / 0.001= 0.000000000155Ф (фарады).
Возведем полученный результат в 12 степень чтобы получить значение емкости в пикофарадах:
C = 0.000000000155 12 = 155пФ.
Полученная нами ёмкость конденсатора 155пф очень мала, обычно такие ёмкости используются в аппаратуре работающей на высоких частотах переменного тока порядка 1 — 600 МГц (мегагерц).
Представьте себе, что мы разрабатываем миниатюрный карманный радиоприемник в котором требуется порядка 30 таких конденсаторов.

Если мы установим в схему 30 разработанных нами конденсаторов, не считая других необходимых радиодеталей, то наш радиоприемник никак не получится миниатюрным. Все дело в том, что объём только наших конденсаторов получится таким, что его никак нельзя будет назвать приемлемым.
Объем одного конденсатора Vc равен Vc = 5см * 5см * 0,1см
Vc = 2,5см в кубе. Тогда объем 30 конденсаторов будет равен:
V = 30 * 2,5 = 75см в кубе.
Что делать, как быть, как уменьшить геометрический объем конденсатора для применения в миниатюрной радиоаппаратуре? Для решения этой проблемы максимально уменьшают расстояние между обкладками, тогда увеличивается емкость и уменьшается геометрический объем конденсатора. Но расстояние уменьшают до определенных пределов иначе конденсатор будет пробиваться даже при низком напряжении подаваемом на конденсатор. В связи с этим на каждом конденсаторе указывается напряжение которое он может выдержать.

Для уменьшения площади обкладок конденсатор делают многослойным состоящим как бы из нескольких параллельно включенных конденсаторов (вспомните формулу параллельного включения конденсаторов).
В качестве диэлектрика в миниатюрных конденсаторах используют тонкие пленки из синтетических материалов, а в качестве обкладок металлическую фольгу, чаще всего из алюминия.



На корпусе конденсатора, обычно, указывается его тип, емкость и рабочее напряжение. Остальные параметры конденсатора определяются из справочников. Емкость конденсатора указывается не так, как на электрических схемах. Например емкость 2,2пФ обозначается 2П2, емкость 1500 пФ — 1Н5, емкость 0,1 мкФ — М1, емкость 2,2 мкФ — 2М2, емкость 10 мкФ — 10М.
У обычных конденсаторов КМ, КД, МБМ и так далее трудно получить большую ёмкость при малых габаритах поэтому были разработаны так называемые электролитические конденсаторы у которых в качестве диэлектрика используется специальная электролитическая жидкость с очень большим Er. Ёмкость таких конденсаторов может достигать сотен тысяч микрофарад. К недостатку таких конденсаторов следует отнести низкое рабочее напряжение (до 500V) и обязательное соблюдение полярности при включении в схему.
Для настройки и подстройки некоторых типов радиоаппаратуры, например радиоприемник или телевизор, применяют специальные конденсаторы с изменяемой ёмкостью.

В зависимости от назначения такие конденсаторы называют «подстроечные» и «конденсаторы переменной емкости».
Емкость переменных и подстроечных конденсаторов изменяется механическим способом, путем изменения расстояния между обкладками или изменения площади пластин. В качестве диэлектрика в таких конденсаторах используется воздух или фарфор.
В заключение следует отметить, что в настоящее время, в связи с бурным развитием радиоэлектроники подстроечные и переменные конденсаторы практически не применяются. Их с успехом заменяют специальные фильтры и полупроводниковые приборы которые не требуют механического изменения параметров.

РАДИОТЕХНИЧЕСКИЕ РАСЧЕТЫ

силовой трансформатор   радиотехнические расчеты   радио калькулятор

                РАДИОТЕХНИЧЕСКИЕ РАСЧЕТЫ

Во время конструирования радиолюбителю приходится производить массу расчетов. Один из самых трудоемких - расчет колебательного контура. Рассмотрим методику такого расчета. 

Как и при любом расчете нам нужны будут исходные данные. Предположим, нам нужно рассчитать частоту колебательного контура для фиксированных значений емкости конденсатора и индуктивности катушки. Допустим, емкость конденсатора равна 10 пикофарадам, индуктивность катушки — 10 микрогенри. По формуле (1 (А)) определяем частоту. Она равна 15900 килогерц (то есть 15,9 Мегагерца). При расчете индуктивности катушки при известных частоте настройки  и емкости контура используем формулу 1(В). Для расчета емкости конденсатора используется формула 1(С).

Статья из журнала «Радио» для упрощенного расчета колебательного контура лежит здесь. Номограмму по расчету числа витков и размера катушек можно скачать по этой ссылке.  Обе статьи в формате DjVu — программу для их чтения можно скачать здесь.

Емкости конденсаторов и сопротивления резисторов имеют стандартный числовой ряд, но иногда требуются нестандартные значения. Как можно выйти из такого положения? Можно взять несколько, например, резисторов и соединить их так, чтобы получить нужное значение.

Пользуясь формулой (2) можно рассчитать величину, которую мы получим в результате параллельного (а), либо последовательного (b) соединения резисторов. При последовательном соединении резисторов их номиналы складываются между собой. Параллельное соединение позволяет получить результирующее сопротивление всегда меньшее, чем номинал наименьшего из соединяемых резисторов. При любом способе соединения резисторов (из рассмотренных) общая мощность рассеяния их увеличивается. Кроме того, при параллельном соединении через резисторы можно пропустить больший ток без их порчи.

Соединение конденсаторов:

При последовательном соединении конденсаторов (формула 3(b)) результирующая емкость будет всегда меньше емкости наименьшего из соединяемых конденсаторов. При параллельном соединении (3(a)) результирующая емкость будет равна сумме емкостей конденсаторов.

При работе на переменном токе иногда приходится рассчитывать реактивные сопротивления катушек индуктивности и конденсаторов.

Реактивное сопротивление катушки можно определить, пользуясь формулой (4), сопротивление конденсатора на переменном токе можно вычислить при помощи формулы (5):

 

В обоих формулах «Pi» — это всем известная математическая константа «Пи», равная (округленно) 3,14.

В заключении хотелось бы обратить ваше внимание на сноски в формулах. Для того, чтобы получить истинное значение при расчетах не забывайте использовать нужные величины!

В дальнейшем мы с вами рассмотрим формулы расчета катушек индуктивности с сердечником и без сердечника.

Формулы для конденсаторов. Энергия конденсатора

Состоит из двух пластин (или обкладок), находящихся одна перед другой и сделанных из проводящего материала. Между пластинами находится изолирующий материал, называемый диэлектриком (рис. 4.1). Простейшими диэлектриками являются воздух, бумага, слюда и т. д.

Рис. 4.1

Зарядка конденсатора

Основным свойством конденсатора является его способность запасать электрическую энергию в виде электрического заряда.
На рис. 4.2(а) изображена схема, в которой конденсатор соединяется через ключ с источником питания. Когда ключ замкнут (рис. 4.2(б)), положительный полюс источника «откачивает» электроны с обкладки А, и она приобретает положительный заряд. Отрицательный полюс источника питания тем временем «поставляет» электроны на обкладку В, в результате чего она приобретает отрицательный заряд, по абсолютной величине равный положительному заряду обкладки А. Такой поток электронов называется током заряда. Он продолжает течь до тех пор, пока напряжение на конденсаторе не сравняется с ЭДС источника питания. В этом случае говорят, что конденсатор полностью заряжен. Электрический заряд обозначается буквой Q, а его величина измеряется в кулонах (Кл).


Рис. 4.2.

Когда конденсатор заряжен, между его обкладками возникает разность потенциалов, а следовательно, и электрическое поле.
Если в момент, когда конденсатор уже зарядился, разомкнуть ключ (рис. 4.2(в)), конденсатор будет хранить заряд. В этом случае внутри диэлектрика между обкладками возникает электрическое поле. При разряде конденсатора через сопротивление нагрузки (рис. 4.2(г)) электрическое ноле исчезает.

Емкость конденсатора

Способность конденсатора накапливать электрический заряд называется емкостью, а величина этой емкости обозначается буквой С и измеряется в фарадах (Ф). Фарада — очень большая единица емкости, и поэтому она практически не используется. Чаще используются дробные единицы:

1 микрофарада (мкФ) = Ф = 10 -6 Ф,

1 пикофарада (пФ) = мкФ = 10 -6 мкФ = 10 -12 Ф.

Емкость конденсатора возрастает с увеличением площади обкладок и убывает с увеличением расстояния между ними.
Например, при возрастании площади обкладок вдвое емкость также увеличивается в два раза. Если же увеличить вдвое расстояние между обкладками, емкость станет вдвое меньше.

Связь заряда, емкости и напряжения

Если конденсатор заряжен до разности потенциалов V , его заряд определяется формулой Q=CV

где С выражается в фарадах, V – в вольтах, а Q – в кулонах. Преобразовав эту формулу, получим:

Энергия заряженного конденсатора

Энергия W, запасенная конденсатором, определяется формулой

где W выражается в джоулях, С – в фарадах, а V — в вольтах.

Параллельное и последовательное соединение конденсаторов

Если два конденсатора, С1 и С2, соединены параллельно (рис. 4.3(а)), результирующая емкость СТ такого соединения равна сумме емкостей этих конденсаторов:

Если конденсаторы соединены последовательно (рис. 4.3(б)), результирующая емкость СТ оказывается меньше емкости любого из конденсаторов я выражается формулой

Например, если С1 = С2, то результирующая емкость СТ последовательного соединения равна половине емкости любого из конденсаторов:

Напряжение на последовательно соединенных конденсаторах

На схеме, показанной на рис. 4.4, конденсаторы С1 и С2 соединены последовательно и подключены к источнику постоянного напряжения VТ. Полное напряжение VТ будет поделено между С1 и С2 таким образом, что на конденсаторе меньшей емкости установится большее напряжение,


Рис. 4.3. Параллельное (а) и последовательное (б) соединение конденсаторов.


и наоборот.

Сумма V1 (напряжения на С1) и V2 (напряжения на С2) всегда равна полному напряжению VТ.
В общем случае, когда несколько конденсаторов, соединенных последовательно, подключено к источнику постоянного тока, напряжение на каждом из конденсаторов обратно пропорционально его емкости. При последовательном соединении двух конденсаторов напряжения на С1 и С2 соответственно равны

Пример 1

Определим результирующую емкость цепи, изображенной на рис. 4.5. Результирующая емкость параллельного соединения равна

С2 + С3 = 10 + 20 = 30 пФ

Поскольку емкость С1 также равна 30 пФ, то результирующая емкость всей цепи равна ½*30 = 15 пФ.



Рис. 4.6. Рис. 4.7.

Пример 2

откуда напряжение на С2 равно 30 – 20 = 10 В.

Рабочее напряжение

Любой конденсатор характеризуется некоторым максимальным напряжением, при превышении которого наступает пробой диэлектрика. Это напряжение называется рабочим, или номинальным, напряжением конденсатора, и подаваемое на конденсатор напряжение ни в коем случае не должно его превышать. При использовании конденсатора в цепях переменного тока амплитудное значение напряжения в цепи также не должно превышать рабочего напряжения конденсатора. Рабочим напряжением для батареи конденсаторов, соединенных параллельно, является наименьшее из рабочих напряжений конденсаторов, входящих в схему, Например, рабочее напряжение для цепи, изображенной на рис. 4.7, равно 25 В.
Для конденсаторов, соединенных последовательно, рабочее напряжение подбирать труднее. Рассмотрим схему на рис. 4.8. Конденсатор С1 (1 мкФ, рабочее напряжение Vраб = 25 В) соединен последовательно с конденсатором С2 (10 мкФ, Vраб = 10 В). Поскольку на конденсаторе С1, обладающем меньшей емкостью, установится большее напряжение, чем на С2, то при расчетах следует прежде всего иметь в виду рабочее напряжение конденсатора С1, равное 25 В. Таким образом, V1 = 25 В. соотношения V1/ V2 = С1/ С2 следует, что

Поскольку рабочее напряжение конденсатора С2 выше, чем V2, рабочее напряжение данной батареи конденсаторов равно 25 + 2,5 = 27,5 В.
Следует заметить, что если бы рабочее напряжение конденсатора было равно, например, 2 В, как показано на рис. 4.9, то он зарядился бы



Рис. 4.8. Рис. 4.9.



Рис. 4.10. Рис. 4.11 . Катушка индуктивности

до уровня рабочего напряжения прежде, чем напряжение на конденсаторе С1 достигло бы 25 В. Вот расчет для этого случая:
V2 = 2 В, тогда.

Следовательно, рабочее напряжение такой батареи будет составлять 20 + 2 = 22 В.

Пример 3

Конденсаторы С1 и С2, изображенные на рис. 4.10, имеют каждый рабочее напряжение 60 В. Какое максимальное напряжение может быть приложено к этой схеме?

Решение
Поскольку на конденсаторе С1 установится более высокое напряжение, чем на конденсаторе С2, то напряжение на нем раньше достигнет уровня рабочего напряжения. При V1 = 60 В

Максимальное напряжение, которое может быть подано на данную схему, составляет 60 + 20 = 80 В.

В этом видео рассказывается о понятии конденсатора:

§ 6. Заряд и разряд конденсатора

Чтобы зарядить конденсатор, надо, чтобы свободные электроны перешли из одной обкладки на другую. Переход электронов с одной обкладки конденсатора на другую происходит под действием напряжения источника по проводам, соединяющим этот источник с обкладками конденсатора.

В момент включения конденсатора зарядов на его обкладках нет и напряжение на нем равно нулю μ с =0. Поэтому зарядный ток определяется внутренним сопротивлением источника r в и имеет наибольшую величину:

I З max =E/ r в.

По мере накопления зарядов на обкладках конденсатора напряжение на нем увеличивается и падение напряжения на внутреннем сопротивлении источника будет равно разности ЭДС источника и напряжения на конденсаторе (Е- μ с). следовательно, зарядный ток

i з =(Е- μ с)/ r в.

Таким образом, с увеличением напряжения на конденсаторе ток заряда снизится и при μ с =Е становится равным нулю. Процесс изменения напряжения на конденсаторе и тока заряда во времени изображен на рис. 1. В самом начале заряда напряжение на конденсаторе резко возрастает, так как зарядный ток имеет наибольшее значение и накопление зарядов на обкладках конденсатора происходит интенсивно. По мере повышения напряжения на конденсаторе зарядный ток уменьшается и накопление зарядов на обкладках замедляется. Продолжительность заряда конденсатора зависит от его емкости и сопротивления цепи, увеличение которых приводит к возрастанию продолжительности заряда. С увеличением емкости конденсатора, возрастает количество зарядов, накапливаемых на его пластинах, а если увеличить сопротивление цепи уменьшится и зарядный ток, а это замедляет процесс накопления зарядов на этих обкладках.

Если обкладки заряженного конденсатора подключить к какому-либо сопротивлению R , то за счет напряжения на конденсаторе будет протекать разрядный ток конденсатора. При разряде конденсатора электронысодной пластины (при их избытке) будут переходить на другую (при их недостатке) и будет продолжается до тех пор, пока потенциалы обкладок не выравняются, т. е. напряжение на конденсаторе станет равным нулю. Изменение напряжения в процессе разряда конденсатора изображено на рис. 2. Ток разряда конденсатора пропорционален напряжению на конденсаторе (i р =μ с /R ), и его изменение во времени подобно изменению напряжения.



В начальный момент разряда напряжение на конденсаторе наибольшее (μ с =Е) и разрядный ток максимальный (I р max =E /R ), так что разряд происходит быстро. При понижении напряжения, ток разряда снижается и процесс перехода зарядов с одной обкладки на другую затормаживается.

Время процесса разряда конденсатора зависит от сопротивления цепи и емкости конденсатора, причем возрастание как сопротивления, так и емкости увеличивает продолжительность разряда. С увеличением сопротивления разрядный ток снижается, замедляется процесс переноски зарядов с одной на другую обкладок; с увеличением емкости конденсатора повышается заряд на обкладках.

Таким образом, в цепи, содержащей конденсатор, ток проходит только в процессе его заряда и разряда, т. е. когда напряжение на обкладках претерпевает изменение во времени. При постоянстве напряжения ток через конденсатор не проходит, т. е. конденсатор не пропускает постоянный ток, так как между его обкладками помещен диэлектрик и в результате этого цепь разомкнута.

При зарядке конденсатора, последний способен накапливать электрическую энергию, потребляя ее от энергоисточника. Накопленная энергия сохраняется определенное время. При разряде конденсатора эта энергия переходит к разрядному резистору, нагревая его, т. е. энергию электрического поля превращается в тепловую. Чем выше емкость конденсатора и напряжение на его обкладках, тем будет больше энергии, запасенной на нем. Энергия электрического поля конденсатора определяется следующим выражением

W=CU 2 /2.

Если конденсатор емкостью 100 мкФ заряжен до напряжения 200 В, то энергия, запасенная в электрическом поле конденсатора, W =100· 10 -6 · 200 2 /2=2 Дж.

Присоединим цепь, состоящую из незаряженного конденсатора емкостью С и резистора с сопротивлением R, к источнику питания с постоянным напряжением U (рис. 16-4).

Так как в момент включения конденсатор еще не заряжен, то напряжение на нем Поэтому в цепи в начальный момент времени падение напряжения на сопротивлении R равно U и возникает ток, сила которого

Рис. 16-4. Зарядка конденсатора.

Прохождение тока i сопровождается постепенным накоплением заряда Q на конденсаторе, на нем появляется напряжение и падение напряжения на сопротивлении R уменьшается:

как и следует из второго закона Кирхгофа. Следовательно, сила тока

уменьшается, уменьшается и скорость накопления заряда Q, так как ток в цепи

С течением времени конденсатор продолжает заряжаться, но заряд Q и напряжение на нем растут все медленнее (рис. 16-5), а сила тока в цепи постепенно уменьшается пропорционально разности — напряжений

Рис. 16-5. График изменения тока и напряжения при зарядке конденсатора.

Через достаточно большой интервал времени (теоретически бесконечно большой) напряжение на конденсаторе достигает величины, равной напряжению источника питания, а ток становится равным нулю — процесс зарядки конденсатора заканчивается.

Процесс зарядки конденсатора тем продолжительней, чем больше сопротивление цепи R, ограничивающее силу тока, и чем больше емкость конденсатора С, так как при большой емкости должен накопиться больший заряд. Скорость протекания процесса характеризуют постоянной времени цепи

чем больше , тем медленнее процесс.

Постоянная времени цепи имеет размерность времени, так как

Через интервал времени с момента включения цепи, равный , напряжение на конденсаторе достигает примерно 63% напряжения источника питания, а через интервал процесс зарядки конденсатора можно считать закончившимся.

Напряжение на конденсаторе при зарядке

т. е. оно равно разности постоянного напряжения источника питания и свободного напряжения убывающего с течением времени по закону показательной функции от значения U до нуля (рис. 16-5).

Зарядный ток конденсатора

Ток от начального значения постепенно уменьшается по закону показательной функции (рис. 16-5).

б) Разряд конденсатора

Рассмотрим теперь процесс разряда конденсатора С, который был заряжен от источника питания до напряжения U через резистор с сопротивлением R (рис. 16-6, Где переключатель переводится из положения 1 в положение 2).

Рис. 16-6. Разряд конденсатора на резистор.

Рис. 16-7. График изменения тока и напряжения при разрядке конденсатора.

В начальный момент, в цепи возникнет ток и конденсатор начнет разряжаться, а напряжение на нем уменьшаться. По мере уменьшения напряжения будет уменьшаться и ток в цепи (рис. 16-7). Через интервал времени напряжение на конденсаторе и ток цепи уменьшатся при мерно до 1% начальных значений и процесс разряда конденсатора можно считать закончившимся.

Напряжение на конденсаторе при разряде

т. е. уменьшается по закону показательной функции (рис. 16-7).

Разрядный ток конденсатора

т. е. он, так же как и напряжение, уменьшается по тому же закону (рис. 6-7).

Вся энергия, запасенная при зарядке конденсатора в его электрическом поле, при разряде выделяется в виде тепла в сопротивлении R.

Электрическое поле заряженного конденсатора, отсоединенного от источника питания, не может долго сохраняться неизменным, так как диэлектрик конденсатора и изоляция между его зажимами обладают некоторой проводимостью.

Разряд конденсатора, обусловленный несовершенством диэлектрика и изоляции, называется саморазрядом. Постоянная времени при саморазряде конденсатора не зависит от формы обкладок и расстояния между ними.

Процессы зарядки и разряда конденсатора называются переходными процессами.

Темы кодификатора ЕГЭ : электрическая ёмкость, конденсатор, энергия электрического поля конденсатора.

Предыдущие две статьи были посвящены отдельному рассмотрению того, каким образом ведут себя в электрическом поле проводники и каким образом — диэлектрики. Сейчас нам понадобится объединить эти знания. Дело в том, что большое практическое значение имеет совместное использование проводников и диэлектриков в специальных устройствах — конденсаторах .

Но прежде введём понятие электрической ёмкости .

Ёмкость уединённого проводника

Предположим, что заряженный проводник расположен настолько далеко от всех остальных тел, что взаимодействие зарядов проводника с окружающими телами можно не принимать во внимание. В таком случае проводник называется уединённым .

Потенциал всех точек нашего проводника, как мы знаем, имеет одно и то же значение , которое называется потенциалом проводника. Оказывается, что потенциал уединённого проводника прямо пропорционален его заряду . Коэффициент пропорциональности принято обозначать , так что

Величина называется электрической ёмкостью проводника и равна отношению заряда проводника к его потенциалу:

(1)

Например, потенциал уединённого шара в вакууме равен:

где — заряд шара, — его радиус. Отсюда ёмкость шара:

(2)

Если шар окружён средой-диэлектриком с диэлектрической проницаемостью , то его потенциал уменьшается в раз:

Соответственно, ёмкость шара в раз увеличивается:

(3)

Увеличение ёмкости при наличии диэлектрика — важнейший факт. Мы ещё встретимся с ним при рассмотрении конденсаторов.

Из формул (2) и (3) мы видим, что ёмкость шара зависит только от его радиуса и диэлектрической проницаемости окружающей среды. То же самое будет и в общем случае: ёмкость уединённого проводника не зависит от его заряда; она определяется лишь размерами и формой проводника, а также диэлектрической проницаемостью среды, окружающей проводник. От вещества проводника ёмкость также не зависит.

В чём смысл понятия ёмкости? Ёмкость показывает, какой заряд нужно сообщить проводнику, чтобы увеличить его потенциал на В . Чем больше ёмкость — тем, соответственно, больший заряд требуется поместить для этого на проводник.

Единицей измерения ёмкости служит фарад (Ф). Из определения ёмкости (1) видно, что Ф = Кл/В.

Давайте ради интереса вычислим ёмкость земного шара (он является проводником!). Радиус считаем приближённо равным км.

МкФ.

Как видите, Ф — это очень большая ёмкость.

Единица измерения ёмкости полезна ещё и тем, что позволяет сильно сэкономить на обозначении размерности диэлектрической постоянной . В самом деле, выразим из формулы (2) :

Следовательно, диэлектрическая постоянная может измеряться в Ф/м:

Так легче запомнить, не правда ли?

Ёмкость плоского конденсатора

Ёмкость уединённого проводника на практике используется редко. В обычных ситуациях проводники не являются уединёнными. Заряженный проводник взаимодействует с окружающими телами и наводит на них заряды, а потенциал поля этих индуцированных зарядов (по принципу суперпозиции!) изменяет потенциал самого проводника. В таком случае уже нельзя утверждать, что потенциал проводника будет прямо пропорционален его заряду, и понятие ёмкости проводника самого по себе фактически утрачивает смысл.

Можно, однако, создать систему заряженных проводников, которая даже при накоплении на них значительного заряда почти не взаимодействует с окружающими телами. Тогда мы сможем снова говорить о ёмкости — но на сей раз о ёмкости этой системы проводников.

Наиболее простым и важным примером такой системы является плоский конденсатор . Он состоит из двух параллельных металлических пластин (называемых обкладками ), разделённых слоем диэлектрика. При этом расстояние между пластинами много меньше их собственных размеров.

Для начала рассмотрим воздушный конденсатор, у которого между обкладками находится воздух

Пусть заряды обкладок равны и . Именно так и бывает в реальных электрических схемах: заряды обкладок равны по модулю и противоположны по знаку. Величина — заряд положительной обкладки — называется зарядом конденсатора .

Пусть — площадь каждой обкладки. Найдём поле, создаваемое обкладками в окружающем пространстве.

Поскольку размеры обкладок велики по сравнению с расстоянием между ними, поле каждой обкладки вдали от её краёв можно считать однородным полем бесконечной заряженной плоскости:

Здесь — напряжённость поля положительной обкладки, — напряженность поля отрицательной обкладки, — поверхностная плотность зарядов на обкладке:

На рис. 1 (слева) изображены векторы напряжённости поля каждой обкладки в трёх областях: слева от конденсатора, внутри конденсатора и справа от конденсатора.

Рис. 1. Электрическое поле плоского конденсатора

Согласно принципу суперпозиции, для результирующего поля имеем:

Нетрудно видеть, что слева и справа от конденсатора поле обращается в нуль (поля обкладок погашают друг друга):

Внутри конденсатора поле удваивается:

(4)

Результирующее поле обкладок плоского конденсатора изображено на рис. 1 справа. Итак:

Внутри плоского конденсатора создаётся однородное электрическое поле, напряжённость которого находится по формуле (4) . Снаружи конденсатора поле равно нулю, так что конденсатор не взаимодействует с окружающими телами.

Не будем забывать, однако, что данное утверждение выведено из предположения, будто обкладки являются бесконечными плоскостями. На самом деле их размеры конечны, и вблизи краёв обкладок возникают так называемые краевые эффекты : поле отличается от однородного и проникает в наружное пространство конденсатора. Но в большинстве ситуаций (и уж тем более в задачах ЕГЭ по физике) краевыми эффектами можно пренебречь и действовать так, словно утверждение, выделенное курсивом, является верным без всяких оговорок.

Пусть расстояние между обкладками конденсатора равно . Поскольку поле внутри конденсатора является однородным, разность потенциалов между обкладками равна произведению на (вспомните связь напряжения и напряжённости в однородном поле!):

(5)

Разность потенциалов между обкладками конденсатора, как видим, прямо пропорциональна заряду конденсатора. Данное утверждение аналогично утверждению «потенциал уединённого проводника прямо пропорционален заряду проводника», с которого и начался весь разговор о ёмкости. Продолжая эту аналогию, определяем ёмкость конденсатора как отношение заряда конденсатора к разности потенциалов между его обкладками:

(6)

Ёмкость конденсатора показывает, какой заряд ему нужно сообщить, чтобы разность потенциалов между его обкладками увеличилась на В. Формула (6) , таким образом, является модификацией формулы (1) для случая системы двух проводников — конденсатора.

Из формул (6) и (5) легко находим ёмкость плоского воздушного конденсатора :

(7)

Она зависит только от геометрических характеристик конденсатора: площади обкладок и расстояния между ними.
Предположим теперь, что пространство между обкладками заполнено диэлектриком с диэлектрической проницаемостью . Как изменится ёмкость конденсатора?

Напряжённость поля внутри конденсатора уменьшится в раз, так что вместо формулы (4) теперь имеем:

(8)

Соответственно, напряжение на конденсаторе:

(9)

Отсюда ёмкость плоского конденсатора с диэлектриком :

(10)

Она зависит от геометрических характеристик конденсатора (площади обкладок и расстояния между ними) и от диэлектрической проницаемости диэлектрика, заполняющего конденсатор.

Важное следствие формулы (10) : заполнение конденсатора диэлектриком увеличивает его ёмкость .

Энергия заряженного конденсатора

Заряженный конденсатор обладает энергией. В этом можно убедиться на опыте. Если зарядить конденсатор и замкнуть его на лампочку, то (при условии, что ёмкость конденсатора достаточно велика) лампочка ненадолго загорится.

Следовательно, в заряженном конденсаторе запасена энергия, которая и выделяется при его разрядке. Нетрудно понять, что этой энергией является потенциальная энергия взаимодействия обкладок конденсатора — ведь обкладки, будучи заряжены разноимённо, притягиваются друг к другу.

Мы сейчас вычислим эту энергию, а затем увидим, что существует и более глубокое понимание происхождения энергии заряженного конденсатора.

Начнём с плоского воздушного конденсатора. Ответим на такой вопрос: какова сила притяжения его обкладок друг к другу? Величины используем те же: заряд конденсатора , площадь обкладок .

Возьмём на второй обкладке настолько маленькую площадку, что заряд этой площадки можно считать точечным. Данный заряд притягивается к первой обкладке с силой

где — напряжённость поля первой обкладки:

Следовательно,

Направлена эта сила параллельно линиям поля (т. е. перпендикулярно пластинам).

Результирующая сила притяжения второй обкладки к первой складывается из всех этих сил , с которыми притягиваются к первой обкладке всевозможные маленькие заряды второй обкладки. При этом суммировании постоянный множитель вынесется за скобку, а в скобке просуммируются все и дадут . В результате получим:

(11)

Предположим теперь, что расстояние между обкладками изменилось от начальной величины до конечной величины . Сила притяжения пластин совершает при этом работу:

Знак правильный: если пластины сближаются , то сила совершает положительную работу, так как пластины притягиваются друг к другу. Наоборот, если удалять пластины alt=»(d_2 > d_1)»> , то работа силы притяжения получается отрицательной, как и должно быть.

С учётом формул (11) и (7) имеем:

Это можно переписать следующим образом:

(12)

Работа потенциальной силы притяжения обкладок оказалась равна изменению со знаком минус величины . Это как раз и означает, что — потенциальная энергия взаимодействия обкладок, или энергия заряженного конденсатора .

Используя соотношение , из формулы (12) можно получить ещё две формулы для энергии конденсатора (убедитесь в этом самостоятельно!):

(13)

(14)

Особенно полезными являются формулы (12) и (14) .

Допустим теперь, что конденсатор заполнен диэлектриком с диэлектрической проницаемостью . Сила притяжения обкладок уменьшится в раз, и вместо (11) получим:

При вычислении работы силы , как нетрудно видеть, величина войдёт в ёмкость , и формулы (12) — (14) останутся неизменными . Ёмкость конденсатора в них теперь будет выражаться по формуле (10) .

Итак, формулы (12) — (14) универсальны: они справедливы как для воздушного конденсатора, так и для конденсатора с диэлектриком.

Энергия электрического поля

Мы обещали, что после вычисления энергии конденсатора дадим более глубокое истолкование происхождения этой энергии. Что ж, приступим.

Рассмотрим воздушный конденсатор и преобразуем формулу (14) для его энергии:

Но — объём конденсатора. Получаем:

(15)

Посмотрите внимательно на эту формулу. Она уже не содержит ничего, что являлось бы специфическим для конденсатора! Мы видим энергию электрического поля , сосредоточенного в некотором объёме .

Энергия конденсатора есть не что иное, как энергия заключённого внутри него электрического поля.

Итак, электрическое поле само по себе обладает энергией. Ничего удивительного для нас тут нет. Радиоволны, солнечный свет — это примеры распространения энергии, переносимой в пространстве электромагнитными волнами.

Величина — энергия единицы объёма поля — называется объёмной плотностью энергии . Из формулы (15) получим:

(16)

В этой формуле не осталось вообще никаких геометрических величин. Она даёт максимально чистую связь энергии электрического поля и его напряжённости.

Если конденсатор заполнен диэлектриком, то его ёмкость увеличивается в раз, и вместо формул (15) и (16) будем иметь:

(17)

(18)

Как видим, энергия электрического поля зависит ещё и от диэлектрической проницаемости среды, в которой поле находится.
Замечательно, что полученные формулы для энергии и плотности энергии выходят далеко за пределы электростатики: они справедливы не только для электростатического поля, но и для электрических полей, меняющихся во времени.

Печать

Конденсатор – электронный компонент, предназначенный для накопления электрического заряда. Способность конденсатора накапливать электрический заряд зависит от его главной характеристики – емкости . Емкость конденсатора (С) определяется как соотношение количества электрического заряда (Q) к напряжению (U).

Емкость конденсатора измеряется в фарадах (F) – единицах, названых в честь британского ученого физика Майкла Фарадея. Емкость в один фарад (1F) равняется количеству заряда в один кулон (1C), создающему напряжение на конденсаторе в один вольт (1V). Вспомним, что один кулон (1С) равняется величине заряда, прошедшего через проводник за одну секунду (1sec) при силе тока в один ампер (1A).

Однако кулон, это очень большое количество заряда относительно того, сколько способно хранить большинство конденсаторов. По этой причине, для измерения емкости обычно используют микрофарады (µF или uF), нанофарады (nF) и пикофарады (pF).

  • 1µF = 0.000001 = 10 -6 F
  • 1nF = 0.000000001 = 10 -9 F
  • 1pF = 0.000000000001 = 10 -12 F

Плоский конденсатор

Существует множество типов конденсаторов различной формы и внутреннего устройства. Рассмотрим самый простой и принципиальный — плоский конденсатор. Плоский конденсатор состоит из двух параллельных пластин проводника (обкладок), электрически изолированных друг от друга воздухом, или специальным диэлектрическим материалом (например бумага, стекло или слюда).

Устройство конденсатора

Заряд конденсатора. Ток

По своему предназначению конденсатор напоминает батарейку, однако все же он сильно отличается по принципу работы, максимальной емкости, а также скорости зарядки/разрядки.

Рассмотрим принцип работы плоского конденсатора. Если подключить к нему источник питания, на одной пластине проводника начнут собираться отрицательно заряженные частицы в виде электронов, на другой – положительно заряженные частицы в виде ионов. Поскольку между обкладками находиться диэлектрик, заряженные частицы не могут «перескочить» на противоположную сторону конденсатора. Тем не менее, электроны передвигаются от источника питания — до пластины конденсатора. Поэтому в цепи идет электрический ток.

В самом начале включения конденсатора в цепь, на его обкладках больше всего свободного места. Следовательно, начальный ток в этот момент встречает меньше всего сопротивления и является максимальным. По мере заполнения конденсатора заряженными частицами ток постепенно падает, пока не закончится свободное место на обкладках и ток совсем не прекратится.

Время между состояниями «пустого» конденсатора с максимальным значением тока, и «полного» конденсатора с минимальным значением тока (т.е. его отсутствием), называют переходным периодом заряда конденсатора.

Напряжение

В самом начале переходного периода зарядки, напряжение между обкладками конденсатора равняется нулю. Как только на обкладках начинают появляться заряженные частицы, между разноименными зарядами возникает напряжение. Причиной этому является диэлектрик между пластинами, который «мешает» стремящимся друг к другу зарядам с противоположным знаком перейти на другую сторону конденсатора.

На начальном этапе зарядки, напряжение быстро растет, потому что большой ток очень быстро увеличивает количество заряженных частиц на обкладках. Чем больше заряжается конденсатор, тем меньше ток, и тeм медленнее растет напряжение. В конце переходного периода, напряжение на конденсаторе полностью прекратит рост, и будет равняться напряжению на источнике питания.

Как видно на графике, сила тока конденсатора напрямую зависит от изменения напряжения.

Формула для нахождения тока конденсатора во время переходного периода:

  • Ic — ток конденсатора
  • C — Емкость конденсатора
  • ?Vc/?t – Изменение напряжения на конденсаторе за отрезок времени

После того как конденсатор зарядился, отключим источник питания и подключим нагрузку R. Так как конденсатор уже заряжен, он сам превратился в источник питания. Нагрузка R образовала проход между пластинами. Отрицательно заряженные электроны, накопленные на одной пластине, согласно силе притяжения между разноименными зарядами, двинутся в сторону положительно заряженных ионов на другой пластине.

В момент подключения R, напряжение на конденсаторе то же, что и после окончания переходного периода зарядки. Начальный ток по закону Ома будет равняться напряжению на обкладках, разделенном на сопротивление нагрузки.

Как только в цепи пойдет ток, конденсатор начнет разряжаться. По мере потери заряда, напряжение начнет падать. Следовательно, ток тоже упадет. По мере понижения значений напряжения и тока, будет снижаться их скорость падения.


Время зарядки и разрядки конденсатора зависит от двух параметров – емкости конденсатора C и общего сопротивления в цепи R. Чем больше емкость конденсатора, тем большее количество заряда должно пройти по цепи, и тем больше времени потребует процесс зарядки/разрядки (ток определяется как количество заряда, прошедшего по проводнику за единицу времени). Чем больше сопротивление R, тем меньше ток. Соответственно, больше времени потребуется на зарядку.

Продукт RC (сопротивление, умноженное на емкость) формирует временную константу ? (тау). За один ? конденсатор заряжается или разряжается на 63%. За пять ? конденсатор заряжается или разряжается полностью.

Для наглядности подставим значения: конденсатор емкостью в 20 микрофарад, сопротивление в 1 килоом и источник питания в 10В. Процесс заряда будет выглядеть следующим образом:

Устройство конденсатора. От чего зависит емкость?

Емкость плоского конденсатора зависит от трех основных факторов:

  • Площадь пластин — A
  • Расстояние между пластинами – d
  • Относительная диэлектрическая проницаемость вещества между пластинами — ?


Чем больше площадь пластин конденсатора, тем больше заряженых частиц могут на них разместится, и тем больше емкость.

Расстояние между пластинами

Емкость конденсатора обратно пропорциональна расстоянию между пластинами. Для того чтобы объяснить природу влияния этого фактора, необходимо вспомнить механику взаимодействия зарядов в пространстве (электростатику).

Если конденсатор не находится в электрической цепи, то на заряженные частицы, расположенные на его пластинах влияют две силы. Первая — это сила отталкивания между одноименными зарядами соседних частиц на одной пластине. Вторая – это сила притяжения разноименных зарядов между частицами, находящимися на противоположных пластинах. Получается, что чем ближе друг к другу находятся пластины, тем больше суммарная сила притяжения зарядов с противоположным знаком, и тем больше заряда может разместится на одной пластине.

Относительная диэлектрическая проницаемость

Не менее значимым фактором, влияющим на емкость конденсатора, является такое свойство материала между обкладками как относительная диэлектрическая проницаемость? . Это безразмерная физическая величина, которая показывает во сколько раз сила взаимодействия двух свободных зарядов в диэлектрике меньше, чем в вакууме.

Материалы с более высокой диэлектрической проницаемостью позволяют обеспечить большую емкость. Объясняется это эффектом поляризации – смещением электронов атомов диэлектрика в сторону положительно заряженной пластины конденсатора.

Поляризация создает внутренне электрическое поле диэлектрика, которое ослабляет общую разность потенциала (напряжения) конденсатора. Напряжение U препятствует притоку заряда Q на конденсатор. Следовательно, понижение напряжения способствует размещению на конденсаторе большего количества электрического заряда.

Ниже приведены примеры значений диэлектрической проницаемости для некоторых изоляционных материалов, используемых в конденсаторах.

  • Бумага – от 2.5 до 3.5
  • Стекло – от 3 до 10
  • Слюда – от 5 до 7
  • Порошки оксидов металлов – от 6 до 20

Номинальное напряжение

Второй по значимости характеристикой после емкости является максимальное номинальное напряжение конденсатора . Данный параметр обозначает максимальное напряжение, которое может выдержать конденсатор. Превышение этого значения приводит к «пробиванию» изолятора между пластинами и короткому замыканию. Номинальное напряжение зависит от материала изолятора и его толщины (расстояния между обкладками).

Следует отметить, что при работе с переменным напряжением нужно учитывать именно пиковое значение (наибольшее мгновенное значение напряжения за период). Например, если эффективное напряжение источника питания будет 50В, то его пиковое значение будет свыше 70В. Соответственно необходимо использовать конденсатор с номинальным напряжением более 70В. Однако на практике, рекомендуется использовать конденсатор с номинальным напряжением не менее в два раза превышающим максимально возможное напряжение, которое будет к нему приложено.

Ток утечки

Также при работе конденсатора учитывается такой параметр как ток утечки. Поскольку в реальной жизни диэлектрик между пластинами все же пропускает маленький ток, это приводит к потере со временем начального заряда конденсатора.

Конденсатор

Конденсатор

Конденсатор

Емкость

Конденсатор — это устройство для хранения разделенного заряда. Нет единого электронного компонента сегодня играет более важную роль, чем конденсатор. Это устройство используется для хранить информацию в памяти компьютера, регулировать напряжение в источниках питания, для создания электрических полей, для хранения электрической энергии, для обнаружения и производить электромагнитные волны и измерять время.Любые два проводника, разделенные изолирующей средой, образуют конденсатор.

А Параллельно-пластинчатый конденсатор состоит из двух пластин, разделенных тонкой изоляционной материал, известный как диэлектрик . В параллельной пластине электроны конденсатора переносятся с одной параллельной пластины на другую.

Мы уже показали, что электрическое поле между пластинами постоянно с величиной E = σ / ε 0 и указывает от положительной пластины к отрицательной.

Следовательно, разность потенциалов между отрицательной и положительной пластинами равна предоставлено

∆U = U пол. — U отриц. = -q ∫ отриц. пол. E · d r = q E d.


При интегрировании d r указывает от отрицательной к положительной пластине в противоположном направлении от E . Следовательно, E · d r = -Edr, и знаки минус отменяют.
Положительный пластина имеет более высокий потенциал, чем отрицательная пластина.

Силовые линии и эквипотенциальные линии для Постоянное поле между двумя заряженными пластинами показано справа. Одна пластина конденсатора удерживает положительный заряд Q, а другая — отрицательный заряд -Q. Заряд Q на пластинах пропорционален потенциалу разность V на двух пластинах. емкость C — пропорциональная константа,

Q = CV, C = Q / V.

C зависит от геометрии конденсатора и типа диэлектрического материала использовал. Емкость параллельного пластинчатого конденсатора с двумя пластинами площадью А расстояние d и отсутствие диэлектрического материала между пластинами составляет

C = ε 0 A / d.

(Электрическое поле E = σ / ε 0 . Напряжение V = Ed = σd / ε 0 . Заряд Q = σA. Следовательно, Q / V = σAε 0 / σd = Aε 0 / d.)
Единица измерения емкости в системе СИ: Кулон / Вольт = Фарад (Ф).
Типичный конденсаторы имеют емкость в диапазоне от пикофарад до микрофарад.

Емкость говорит нам, сколько заряда устройство хранит для данного Напряжение. Диэлектрик между проводниками увеличивает емкость конденсатор. Молекулы диэлектрического материала поляризованы в поле между двумя проводниками. Весь отрицательный и положительный заряд диэлектрик смещен на небольшую величину относительно друг друга.Этот приводит к эффективному положительному поверхностному заряду на одной стороне диэлектрика. и отрицательный поверхностный заряд на другой стороне диэлектрика. Эти эффективные поверхностные заряды на диэлектрике создают электрическое поле, которое противостоит полю, создаваемому поверхностными зарядами на проводниках, и, таким образом, снижает напряжение между проводниками. Чтобы поддерживать напряжение, больше заряда необходимо надеть на проводники. Таким образом, конденсатор сохраняет больше заряда для заданное напряжение.Диэлектрическая проницаемость κ — это отношение напряжения V 0 между проводниками без диэлектрика до напряжение V с диэлектриком, κ = V 0 / V, для данного количества заряда Q на проводниках.

На диаграмме выше такое же количество заряда Q на проводников приводит к меньшему полю между пластинами конденсатора с диэлектрик. Чем выше диэлектрическая проницаемость κ, тем больше заряда может хранить конденсатор при заданном напряжении.Для параллельной пластины конденсатор с диэлектриком между пластинами, емкость

C = Q / V = ​​κQ / V 0 = κε 0 A / d = εA / d,

, где ε = κε 0 . Статическая диэлектрическая проницаемость любого материала всегда больше 1.

Типичные диэлектрические постоянные

Материал Диэлектрическая проницаемость
Воздух 1.00059
Силикат алюминия от 5,3 до 5,5
Бакелит 3,7
Пчелиный воск (желтый) 2,7
Бутилкаучук 2,4
Formica XX 4,00
Германий 16
Стекло от 4 до 10
Гуттаперча 2.6
Масло Halowax 4,8
Кел-Ф 2,6
Люцит 2,8
Слюда от 4 до 8
Микарта 254 от 3,4 до 5,4
Майлар 3,1
Неопреновый каучук 6,7
Нейлон 3.00
Материал Диэлектрическая проницаемость
Бумага 1,5 по 3
Парафин от 2 до 3
Оргстекло 3,4
Полиэтилен 2,2
Полистирол 2,56
Фарфор 5-7
Стекло Pyrex 5.6
Кварц от 3,7 до 4,5
Силиконовое масло 2,5
Стеатит от 5,3 до 6,5
Титанат стронция 233
тефлон 2,1
Tenite от 2,9 до 4,5
Вакуум 1,00000
Вазелин 2.16
Вода (дистиллированная) от 76,7 до 78,2
Дерево от 1,2 до 2,1

Если диэлектрик с диэлектрической проницаемостью κ вставляется между пластинами параллельной пластины конденсатора, а напряжение поддерживается постоянным аккумулятором, заряд Q на пластинах увеличивается в κ раз. Батарея перемещает больше электронов с положительной пластины на отрицательную. Величина электрического поля между пластинами E = V / d остается неизменной. тем же.

Если диэлектрик вставлен между пластинами параллельной пластины конденсатор, и заряд на пластинах остается прежним, потому что конденсатор отключается от АКБ, то напряжение V уменьшается в раз из κ, а электрическое поле между пластиной E = V / d уменьшается в 2 раза. κ.


Энергия, запасенная в конденсаторе

Энергия U, запасенная в конденсаторе, равна работе W сделано при разделении заряды на проводниках.Чем больше заряда уже накоплено на пластинах, тем необходимо проделать больше работы по разделению дополнительных сборов из-за сильного отталкивание между одноименными зарядами. При заданном напряжении требуется бесконечно малое объем работы ∆W = V∆Q для отделения дополнительной бесконечно малой суммы заряда ∆Q.
(Напряжение V — это количество работы на единицу заряда.)
Поскольку V = Q / C, V увеличивается линейно с Q. Общая работа, проделанная при зарядке конденсатора является

W = ∫ 0 Qf VdQ = ∫ 0 Qf (Q / C) dQ = ½ (Q f 2 / C) = ½VQ F = V в среднем Q f
Используя Q = CV, мы также можем написать U = ½ (Q 2 / C) или U = ½CV 2 .

Задача:

Каждая ячейка памяти компьютера содержит конденсатор для хранения заряда. Сохраняемый или не сохраняемый заряд соответствует двоичным цифрам 1 и 0. Для более плотной упаковки ячеек в траншейных конденсаторах часто используются пластины конденсатора установлены вертикально вдоль стенок траншеи выгравирован на кремниевом чипе. Если у нас емкость 50 фемтоФарад = 50 * 10 -15 F и каждая пластина имеет площадь 20 * 10 -12 м 2 (траншеи микронного размера), что такое разделение пластин?

Решение:

  • Рассуждение:
    Емкость параллельного пластинчатого конденсатора с двумя пластинами площадью А расстояние между пластинами равно d и отсутствие диэлектрического материала C = ε 0 A / d.
  • Детали расчета:
    C = ε 0 A / d, d = ε 0 A / C = (8,85 * 10 -12 * 20 * 10 -12 / (50 * 10 -15 )) м = 3,54 * 10 -9 м.
    Типичные атомные размеры порядка 0,1 нм, поэтому траншея находится на порядка 30 атомов в ширину.

Ссылка: PhET Конденсаторная лаборатория (базовая)


Для любого изолятора существует максимальное поддерживаемое электрическое поле без ионизации молекул.Для конденсатора это означает, что есть максимально допустимое напряжение, которое может быть приложено к проводникам. Этот максимальное напряжение зависит от диэлектрика в конденсаторе. Соответствующие максимальное поле E b называется диэлектрической прочностью материала. Для более сильных полей конденсатор « выходит из строя » (аналогично коронный разряд) и обычно разрушается. Большинство конденсаторов, используемых в электрических схемы имеют как емкость, так и номинальное напряжение.Это напряжение пробоя V b относится к диэлектрической прочности E b . Для параллельной пластины Конденсатор у нас б В = б д.

Материал Диэлектрическая прочность (В / м)
Воздух 3 * 10 6
Бакелит 24 * 10 6
Неопреновый каучук 12 * 10 6
Нейлон 14 * 10 6
Бумага 16 * 10 6
Полистирол 24 * 10 6
Стекло Pyrex 14 * 10 6
Кварц 8 * 10 6
Силиконовое масло 15 * 10 6
Титанат стронция 8 * 10 6
тефлон 60 * 10 6

Последовательные или параллельные конденсаторы

Конденсатор — это устройство для хранения разделенного заряда и, следовательно, хранения электростатическая потенциальная энергия.Цепи часто содержат более одного конденсатора.

Рассмотрим два конденсатора, подключенных параллельно , как показано справа

Когда батарея подключена, электроны будут течь до тех пор, пока потенциал точки А не станет равным. такой же, как потенциал положительной клеммы аккумулятора и потенциал точки B равен потенциалу отрицательной клеммы аккумулятора. Таким образом, разность потенциалов между пластинами обоих конденсаторов составляет V A — V B = V bat .Имеем C 1 = Q 1 / V bat и C 2 = Q 2 / V bat , где Q 1 — заряд конденсатора C 1 , а Q 2 — заряд конденсатора С 2 . Пусть C — эквивалентная емкость двух конденсаторов. параллельно, то есть C = Q / V bat , где Q = Q 1 + Q 2 . Тогда C = (Q 1 + Q 2 ) / V bat = C 1 + C 2 .

Для конденсаторов, включенных параллельно, емкости складываются. Более двух конденсаторы у нас
C = C 1 + C 2 + C 3 + С 4 + ….


Рассмотреть два конденсатора серии , как показано справа.
Пусть Q представляют собой общий заряд на верхней пластине C 1 , который затем вызывает заряд -Q на его нижней пластине. Заряд на нижней пластине C 2 будет -Q, что, в свою очередь, индуцирует заряд + Q на своей верхней пластине, как показано.
Пусть V 1 и V 2 представляют собой разности потенциалов между пластины конденсаторов С 1 и С 2 соответственно.
Затем V 1 + V 2 = V bat , или (Q / C 1 ) + (Q / C 2 ) = Q / C, или (1 / C 1 ) + (1 / C 2 ) = 1 / C.

Более двух конденсаторы последовательно имеем
1 / C = 1 / C 1 + 1 / C 2 + 1 / С 3 + 1 / С 4 +….
где C эквивалентно емкость двух конденсаторов.
Для конденсаторов последовательно величина, обратная их эквивалентной емкости, равна сумме обратных величин их индивидуальные емкости.

Задача:

Какую общую емкость можно получить, подключив 5 мкФ и 8 мкФ конденсатор вместе?

Решение:

  • Рассуждение:
    Мы можем подключать конденсаторы последовательно или параллельно.
    Чтобы получить наибольшую емкость, мы необходимо подключить конденсаторы параллельно.
    Чтобы получить наименьшую емкость, мы должны подключить конденсаторы последовательно.
  • Детали расчета:
    Параллельное подключение конденсаторов:
    C наибольший = (5 + 8) мкФ = 13 мкФ.
    Последовательное соединение конденсаторов.
    1 / C наименьшее = (1/5 + 1/8) (мкФ) -1 = 13 / (40 мкФ) = 0,325 / мкФ.
    C наименьший = 40/13 мкФ = 3.077 мкФ.

Модуль 5: Вопрос 2:

(a) Конденсатор с параллельными пластинами изначально имеет напряжение 12 В и остается подключенным к батарее. Если теперь расстояние между пластинами увеличено вдвое, что бывает?
(b) Конденсатор с параллельными пластинами первоначально подключается к батарее, а пластины удерживают заряд ± Q. Затем аккумулятор отключается. Если расстояние между пластинами равно теперь вдвое, что происходит?

Подсказка: аккумулятор является зарядным насосом.Может качать заряд с одной пластины к другому, чтобы поддерживать постоянную разность потенциалов.
Нет батареи <--> нет зарядного насоса. Заряд не может перемещаться с одной пластины на Другие.

Обсудите это со своими однокурсниками на дискуссионном форуме!

Суперконденсатор


Суперконденсатор основан на идеальной модели хранения. Идеализированная модель хранения воспроизводит простую модель хранения, которая предполагает плоскую кривую разряда, потому что напряжение питания остается в основном постоянным во время цикла разряда.Для этой модели вам нужно ввести только номинальную мощность в ампер-часах. HOMER использует это как фактическую емкость хранилища. Например, некоторые высокопроизводительные литий-ионные батареи можно хорошо смоделировать с помощью модели идеального хранилища. Чтобы узнать больше об этой модели, обратитесь к статье об идеальной модели хранения.

Свойства суперконденсатора

В этом разделе содержится информация о номинальном напряжении, номинальной емкости и максимальном токе заряда и разряда идеализированной батареи.Энергия (джоули), запасенная в суперконденсаторе, может быть рассчитана по следующей формуле:

эджоулей = 1/2 C V2 (1)

В приведенном выше уравнении E — запасенная энергия в джоулях, C — емкость в фарадах, а V — напряжение. Вы можете указать номинальную емкость как мощность (в ватт-часах), деленную на номинальное напряжение, чтобы получить эффективную емкость в ампер-часах. Это не то же самое, что вычисление зарядной емкости конденсатора (фарады x вольт), которая дает правильную общую энергетическую емкость.Уравнение (1), приведенное выше, учитывает пропорциональное уменьшение напряжения по мере разряда конденсатора. Расчет номинальной входной мощности (в Ач) для HOMER выглядит следующим образом:

NAh = эджоулей / V / 3600 = 1/2 C V / 3600 (2)

В этом уравнении NAh — номинальная мощность в ампер-часах. Вы делите на 3600, чтобы преобразовать джоули в ватт-часы (или, что то же самое, из кулонов или ампер-секунд в ампер-часы).

Входы на весь срок службы

Вы можете указать срок службы батареи суперконденсаторов с помощью входов срока службы, которые отображаются сразу под таблицей затрат.Вы можете указать срок службы по календарным годам или по производительности в кВтч. Если указаны и время (годы), и производительность (кВтч), замена компонентов происходит в соответствии с той, которая происходит первой.

Специальные входы суперконденсатора

В разделе «Ввод для конкретного сайта» можно вводить параметры, которые могут изменяться от проекта к проекту. Вход Initial State of Charge (в процентах) устанавливает состояние заряда аккумулятора в начале моделирования.Вход Minimum State of Charge (в процентах) устанавливает нижний предел уровня заряда.

Разность потенциалов и емкость


Разность потенциалов и емкость

Следующий текст используется только для обучения, исследований, стипендий, образовательных целей и информационных целей в соответствии с принципами добросовестного использования.

Мы благодарим авторов текстов и исходный веб-сайт, который дает нам возможность поделиться своими знаниями

Физика

Разница потенциалов (стр.d.) между двумя точками — это работа, выполняемая по переносу заряда в 1 кулон из одной точки в другую *.
Единица измерения разности потенциалов — Вольт (обозначение В)

Вольт
Разница потенциалов между двумя точками составляет один вольт, если один Джоуль работы выполняется при переносе заряда в один кулон из одной точки в другую.

Более распространенным (но менее правильным) словом для термина «разность потенциалов» является «напряжение».
Обратите внимание, что разность потенциалов всегда находится между двумя точками.

Взаимосвязь между работой, зарядом и напряжением

Работа = Напряжение × Заряд:

См. Решенные задачи 1–6, стр. 236.
В частности, следует попробовать задачи 5 и 6, но, возможно, решение № 5 следует оставить до 6-го года обучения. Затем попробуйте 1 — 8, стр. 237.

Потенциал в точке *
Потенциал в точке относится к работе, проделанной по переносу положительного заряда из этой точки на землю.

Завершение программы Cert Physics Syllabus: потенциальная разница

Содержание

Глубина обработки

Деятельность

СТС

Определение разности потенциалов:
выполненных работ на единицу заряда на
переводить заряд с одной точки
к другому.
Определение вольт.
Понятие нулевого потенциала.

Соответствующие расчеты.


Емкость *
Емкость обозначена символом C
Не путайте «Емкость» (символ C) с «Зарядом» (символ Q).

Конденсатор — электрическое устройство, используемое для хранения заряда

Емкость проводника — это отношение заряда проводника к его потенциалу *.

Единица измерения емкости — Фарада; символ F.
(Это похоже на определение сопротивления, которое мы встретим в следующей главе.)

Конденсатор с параллельными пластинами *
Рассмотрим две противоположно заряженные параллельные пластины, как показано *

Из диаграммы мы видим, что емкость будет увеличиваться, если общая площадь между пластинами (A) увеличивается, или если расстояние между пластинами (d) уменьшается.

Следовательно, C µ A и C µ Þ C µ Þ C = k

Пропорциональная константа оказывается e (помните, что мы столкнулись с этим в предыдущей главе как часть закона Кулона) и представляет собой диэлектрическую проницаемость среды между двумя пластинами.

Þ

См. Рабочие примеры 11 и 12, стр. 241. Затем попробуйте вопросы 1–5, стр. 243

Для демонстрации факторов, влияющих на емкость параллельного пластинчатого конденсатора *


  • Подключите две параллельные пластины к цифровому мультиметру (DMM), установленному для считывания емкости.Обратите внимание на емкость.
  • Увеличьте расстояние между ними — обратите внимание, что емкость уменьшается.
  • Слегка отодвиньте одну пластину в сторону (уменьшая площадь перекрытия) — обратите внимание, что емкость уменьшается.
  • Поместите разные плиты изоляционного материала между пластинами — обратите внимание, что емкость самая низкая, когда между пластинами нет ничего (воздуха) *.

Две формулы для энергии, запасенной в конденсаторе *

Вт =

Чтобы показать, что заряженный конденсатор накапливает энергию

  • Установите, как показано.
  • Включите переключатель, чтобы зарядить конденсатор.
  • Извлеките аккумулятор и соедините клеммы вместе, чтобы «замкнуть» цепь.
  • Лампа будет мигать, когда конденсатор разряжается, показывая, что он накапливает энергию.

Общие области применения конденсаторов

  • «Вспышка» на фотоаппарате
  • Сглаживание колебаний постоянного тока
  • Допускает переменный ток (a.c.), чтобы проходить через него, но блокирует постоянный ток (d.c.) *.
  • Используется в цепях, чтобы пропускать только переменный ток определенной частоты *

Учебный план по физике: емкость

Содержание

Глубина обработки

Деятельность

СТС

Конденсаторы и
емкость

Определение: C = Q / V
Единица измерения емкости.

Конденсатор с параллельными пластинами.
Использование

Демонстрация того, что емкость зависит от общей площади, расстояния между пластинами, и природы диэлектрика.
Соответствующие расчеты.

Общее использование конденсаторов:
• тюнинг магнитолы
• вспышки
• сглаживание
• фильтрация.

Энергия, накопленная в конденсаторе.

Использование W = ½ CV 2

Конденсаторы — проведите а.c. но не
Округ Колумбия.

Конденсатор зарядный — разряд через лампу или низковольтный постоянный ток. мотор.

Соответствующие расчеты.
Демонстрация.


Дополнительный кредит
* Определение потенциальной разницы
Примечание:
Если заряд +1 переходит в положительную точку, необходимо работать, чтобы получить его, но если он переходит в отрицательную точку, то это (заряд +1).
Это похоже на утверждение, что камень на вершине утеса имеет потенциальную энергию, скажем, 100 Джоулей.
Чтобы поднять камень наверх, вы должны потратить 100 Джоулей работы на скале, или, альтернативно, если бы камень упал на землю , сделал бы 100 Джоулей работы.

* W = V Q
Теперь, если вы знаете, сколько гравитационной энергии имеет камень (из формулы для гравитационной потенциальной энергии E = mgh), вы можете рассчитать скорость, с которой он ударится о землю (из формулы для кинетической энергии E = ½ mv2).
От сохранения энергии; Потенциальная энергия = кинетическая энергия: mgh = ½ мв2

Как камень будет ускоряться к земле, так и положительный заряд будет ускоряться в сторону отрицательной точки.
Фактически, если вы знаете, сколько потенциальной энергии должно было иметь заряд в начале, то есть потенциал между двумя точками (из формулы W = QV), вы можете рассчитать его скорость, когда он достигнет второй точки (если вы знаете его массу). снова используя
Потенциальная энергия = кинетическая энергия или QV = ½ мв2

* Потенциал в точке
Помните выше, что мы отметили, что разность потенциалов всегда находится между двумя точками?
Очень часто нам нужно знать, насколько сложно передать положительный заряд от земли к положительному объекту.
Поскольку это происходит так часто, мы отказываемся от термина «разность потенциалов» и просто называем разность потенциалов между землей и объектом «потенциалом объекта» — это также известно как «потенциал в точке».
Из этого должно быть очевидно, что чем больше положительного заряда у объекта, тем больше работы потребуется для того, чтобы подвести к нему еще один положительный заряд, и поэтому потенциал в этой точке будет больше.

Теперь вы можете бросить вызов себе и объяснить, как можно использовать электроскоп с золотым листом для сравнения разности потенциалов между одним объектом и землей по сравнению с другим объектом и землей (большой совет см. На стр. 235).

* Емкость
Представьте емкость с водой.
Количество воды, которое может вместить контейнер (его вместимость), будет зависеть, помимо прочего, от того, насколько быстро поднимется уровень воды при наливании воды; если уровень воды быстро поднимается, это означает, что емкость должна быть довольно узкой и, следовательно, не может содержать много воды.
Мы могли бы сделать вывод, что контейнер имел небольшую «емкость».

Электрический конденсатор можно сравнить с этим резервуаром для воды, и скорость, с которой увеличивается потенциал конденсатора, дает нам представление о том, сколько заряда конденсатор может удерживать; если небольшой заряд на нем значительно увеличивает его потенциал, то его емкость должна быть небольшой.
Помните, что «значительно повышает свой потенциал» означает, что необходимо проделать гораздо больше работы, чтобы повысить его эффективность.

Так, например, если емкость конденсатора составляет 2 фарада, то установка на него заряда в 6 кулонов увеличит его потенциал на 3 вольта (от C = Q / V, поэтому V = Q / C).

Однако, если бы емкость конденсатора составляла 200 фарад, установка на него заряда в 6 кулонов повысила бы его потенциал только на 0,03 вольт.

* Определение Емкость
Если, когда вас попросят определить емкость, вы ответите, что « используется для хранения заряда» , вы получите нулевые оценки, потому что это не определение.

* C = Q / V
Формулу C = Q / V легче запомнить, если переставить ее как Q = CV

.

* Конденсатор с параллельными пластинами

Потенциал положительно заряженного объекта является показателем того, сколько работы потребуется сделать, чтобы подвести к нему другой положительно заряженный объект.
Следовательно, чем больше положительного заряда на объекте, тем больше будет его потенциал.

Однако если мы поднесем отрицательно заряженный объект близко к положительно заряженному объекту, их электрические поля будут иметь тенденцию нейтрализовать друг друга, и тогда станет легче подвести больше положительного заряда к исходному — положительно заряженному — объекту.

Это означает, что потенциал исходного объекта уменьшается, просто поднося к нему противоположно заряженный проводник.

Это принцип, на котором основан конденсатор с параллельными пластинами.

* Рассмотрим две противоположно заряженные параллельные пластины, как показано на рисунке

Ключ к этому — помнить, что чем больше их электрические поля нейтрализуют друг друга, тем больше будет емкость системы.

* Две формулы для энергии, запасенной в конденсаторе
W = ½ CV2, W = Q2 / 2C
Вам не нужно знать, как выводить эти формулы, но на самом деле это очень просто, и, по крайней мере, для меня легче вывести их с нуля, чем запомнить.
Их вывод включает формулы W = QV и C = Q / V.
Я могу показать вам доску через 5 минут, если вы спросите меня.
Обратите внимание: если вы начнете с Q = CV и замените это на W = QV, вы получите W = CV без коэффициента 1/2!
Чтобы объяснить, откуда берется множитель ½, необходимо пройтись по выводам, так что продолжайте — спросите меня!
Кстати, учебники не объясняют эту явную аномалию.

* Допускает переменный ток (a.c.), чтобы пройти через него, но блокирует постоянный ток (d.c.) *.

На первой диаграмме (постоянный ток), когда цепь замкнута, происходит внезапное движение электронов, но поскольку цепь не замкнута, дальнейшего потока нет.
На второй диаграмме пластины непрерывно заряжаются и разряжаются; время (и, следовательно, частота) зависит от размера пластин (см. следующий пункт).
Строго говоря, ток не «проходит» через конденсатор, а просто действует так, как будто он есть.

* Используется в цепях, чтобы пропускать только переменный ток определенной частоты
Это, в свою очередь, используется в радиоприемниках для приема только определенных частот , соответствующих радиоволнам, исходящим от определенной радиостанции.
Часть электронной схемы, отвечающая за это, называется RC-цепью (сопротивление, емкость).
При смене станций в радио вы действительно меняете емкость этой части схемы.
Как это работает?
Чем больше размер конденсатора, тем больше времени требуется для разряда (и зарядки). Теперь, если время, необходимое для зарядки и разрядки, соответствует частоте переменного тока, который, в свою очередь, передает звуковую информацию от радиостанции, тогда этот сигнал будет пропущен и в конечном итоге достигнет динамиков.

* Для демонстрации факторов, влияющих на емкость параллельного пластинчатого конденсатора
Примечание. Альтернативная демонстрация приведена на странице 242, но понять, что происходит, до смешного сложно — учебник не объясняет это полностью.
Описанная демонстрация вполне адекватна и очень проста.

* Поместите разные плиты изоляционного материала между пластинами — обратите внимание, что емкость самая низкая, когда между пластинами нет ничего (или воздуха).
Это может показаться запутанным, поэтому, если вас это беспокоит, попросите меня объяснить, почему.

Экзаменационные вопросы

  • [2005] [2004] [2009] [2002 OL]

Определите разность потенциалов.

Назовите прибор, используемый для измерения разности потенциалов.

  • [2009] [2008] [2004] [2002 OL]

Определите емкость.

Назовите электрический компонент, представленный на схеме.

  • [2006 год] [2006 год] [2007 год] [2010]

Дайте одно использование конденсатора.

Какое из следующих устройств настраивается при настройке на радиостанцию?
Трансформатор, диод, конденсатор, реостат

Перечислите факторы, влияющие на емкость конденсатора с параллельными пластинами.

Как бы вы продемонстрировали, что емкость конденсатора с параллельными пластинами зависит от расстояния между его пластинами?

Какой положительный заряд сохраняется на конденсаторе 5 мкФ при подключении к напряжению 120 В постоянного тока? поставка?

  • [2006]
  • Пластины заполненного воздухом конденсатора с параллельными пластинами имеют общую площадь 40 см2 и расположены на расстоянии 1 см друг от друга. Конденсатор подключен к источнику постоянного тока 12 В. поставка. Рассчитайте емкость конденсатора.

(Проницаемость свободного пространства = 8.85 × 10-12 Фм-1)

  • Рассчитайте величину заряда на каждой пластине.
  • Каков чистый заряд конденсатора?

Опишите эксперимент, демонстрирующий, что конденсатор может накапливать энергию.

  • Конденсатор подключен к переключателю, батарее и лампочке, как показано на схеме. Когда переключатель переводится из положения A в положение B, лампочка загорается на короткое время.
  • Что происходит с конденсатором, когда переключатель находится в положении A?
  • Почему лампочка загорается, когда переключатель находится в положении B?
  • Когда переключатель находится в положении A, конденсатор имеет заряд 0.6 C, рассчитайте его емкость.

Рассчитайте энергию, запасенную в конденсаторе 5 мкФ, когда к нему приложена разность потенциалов 20 В.

Конденсатор емкостью 100 мкФ заряжается до разности потенциалов 20 В. Какая энергия хранится в конденсаторе?

Сколько энергии хранится в конденсаторе емкостью 100 мкФ, когда он заряжается до разности потенциалов 12 В?

Способность конденсатора накапливать энергию — основа дефибриллятора.Во время сердечного приступа камеры сердца не могут перекачивать кровь, потому что их мышечные волокна сокращаются и расслабляются беспорядочно. Чтобы спасти жертву, необходимо потрясти сердечную мышцу, чтобы восстановить нормальный ритм.
Дефибриллятор используется для разряда сердечной мышцы.
Конденсатор на 64 мкФ в дефибрилляторе заряжается до разности потенциалов 2500 В.
Конденсатор разряжается за 10 миллисекунд через электроды, прикрепленные к груди пострадавшего от сердечного приступа.

  • Рассчитайте заряд, накопленный на каждой пластине конденсатора.
  • Рассчитайте энергию, запасенную в конденсаторе.
  • Рассчитайте среднюю мощность, генерируемую при разряде конденсатора.

  • [2004]
  • На принципиальной схеме показан конденсатор емкостью 50 мкФ, соединенный последовательно с резистором, батареей на 6 В и переключателем.

Разность потенциалов на конденсаторе равна 2.Первоначально 24 В при токе 80 мкА.
Рассчитайте заряд конденсатора в этот момент.

  • Рассчитайте энергию, запасенную в конденсаторе, когда он полностью заряжен.
  • Опишите, что происходит в цепи при напряжении 6 В постоянного тока. источник питания заменен на 6 В переменного тока. поставка.

На схеме A показан конденсатор, подключенный к лампочке, и напряжение 12 В переменного тока. поставка.
На схеме B показан тот же конденсатор, подключенный к лампочке, но подключенный к напряжению 12 В d.c. поставка.
Что происходит в каждом случае, когда переключатель замкнут? Поясните свой ответ.

Решения для экзаменов

  • Разность потенциалов между двумя точками — это работа, совершаемая при переносе заряда в 1 кулон из одной точки в другую.
  • Вольтметр
  • Емкость проводника — это отношение заряда проводника к его потенциалу.
  • А конденсатор
  • Накопитель заряда / (радио) настройка / сглаживание / накопление энергии / вспышки для фотоаппаратов, зарядное устройство для телефона, блоки d.c.
  • Конденсатор
  • Общая площадь пластин, расстояние друг от друга, диэлектрическая проницаемость между пластинами.
  • Подключите две параллельные пластины к цифровому мультиметру (DMM) для считывания емкости.

Обратите внимание на емкость.
Увеличьте расстояние между ними — обратите внимание, что емкость уменьшается.

  • Q = CV = (5 × 10–6) (120)

Q = 6.0 × 10–4 C

C = [(8.85 × 10-12) (40 × 10-4)] / (0,01)
C = 3,54 × 10-12 F

Q = (3,54 x 10-12) (12) = 4,2 (5) x 10-11 C

  • Ноль
  • Установите, как показано.
  • Включите переключатель, чтобы зарядить конденсатор.
  • Извлеките аккумулятор и соедините клеммы вместе, чтобы «замкнуть» цепь.
  • Лампа будет мигать, когда конденсатор разряжается, показывая, что он накапливает энергию.
  • Заряжается (на пластинах накапливается заряд).
  • Конденсатор разряжается, и через лампочку на короткое время проходит ток.
  • C = Q / V = ​​0,6 / 6 = 0,1 Фарад
  • E = ½ CV2 = ½ (5 x 10-6) (20) 2 = 1,0 x 10-3 Дж
  • E = ½CV2 Þ E = ½ (100 × 10-6) (20) 2 = 0,02 Дж
  • E = ½ CV2 = ½ (100 × 10-6) (12) 2 = 7,2 × 10-3 Дж
  • C = Q / v Þ q = CV Þ q = (64 × 10-6) (2500) Þ q = 0.16 C
  • E = ½ CV2 = ½ (64 × 10-6) (2500) 2 = 200 Дж
  • P = Вт / t = (200) / (10 × 10-3) = 20000 Вт
  • C = Q / V Þ Q = CV = (50 × 10−6) (2,24) = 1,12 × 10-4 C
  • E = ½ CV2 = ½ (50 × 10−6) (6) 2 = 9 × 10−4 Дж

Обратите внимание, что здесь мы используем 6 В в качестве разности потенциалов на конденсаторе, потому что, хотя разность потенциалов вначале составляет 2,24 В, когда конденсатор полностью заряжен, требуется максимальное количество энергии для увеличения заряда.

  • Ток будет течь постоянно.
  • На схеме A лампа горит, потому что конденсатор «проводит» переменный ток. (он постоянно заряжается и разряжается).

На схеме B лампа не горит, потому что конденсатор не проводит постоянный ток.

Источник: http://www.thephysicsteacher.ie/LC%20Physics/Student%20Notes/20.%20Potential%20Difference%20and%20Capacitance.doc

Ссылка на веб-сайт: http://www.thephysicsteacher.ie

Автор: не указан в исходном документе текста выше

Разность потенциалов и емкость

Разность потенциалов и емкость

На главную

Разность потенциалов и емкость

Это правильное место, где вы найдете ответы на свои вопросы, например:

Кто? Какие ? Когда ? Где ? Почему ? Который ? Как ? Что означают разность потенциалов и емкость? Что означают разность потенциалов и емкость?


Примечания по физике разности потенциалов и емкости


Аланпедия.com с 1998 г. год от года новые сайты и инновации

На главную Заявление об ограничении ответственности Свяжитесь с нами

Емкость

  • • Что такое емкость?
  • • Диэлектрик.
  • • диэлектрическая проницаемость.
  • • Диэлектрическая прочность и максимальное рабочее напряжение.
  • • Расчет заряда конденсатора.

Емкость

Количество энергии, которое может хранить конденсатор, зависит от величины или ЕМКОСТИ конденсатора. Емкость (символ C) измеряется в основной единице FARAD (символ F). Один фарад — это величина емкости, которая может хранить 1 кулон (6,24 x 10 18 электронов), когда он заряжен до напряжения 1 вольт. Однако Фарада слишком большая единица для использования в электронике, поэтому следующие единицы емкости более полезны.

Дополнительный блок Аббревиатура Стандартное обозначение
мкФ мкФ х 10 -6
нано Фарады нФ х 10 -9
пик Фарад пФ х 10 -12

Помните, однако, что при решении задач, связанных с емкостью, используемые формулы и значения должны быть в основных единицах измерения: фарадах, вольтах и ​​т. Д.Поэтому при вводе значения 0,47 нФ, например, в формулу (или ваш калькулятор), его следует вводить в фарадах, используя версию стандартной формы для инженерных обозначений: 0,47 x 10 -9 (Загрузите буклет «Советы по математике», чтобы узнать больше Информация).

Емкость зависит от четырех вещей;

1.Площадь плит

2. Расстояние между пластинами

3. Тип диэлектрического материала

4. Температура

Из этих четырех наименьшее влияние на большинство конденсаторов оказывает температура.Стоимость большинства конденсаторов довольно стабильна в «нормальном» диапазоне температур.

Значения конденсатора могут быть фиксированными или переменными. Большинство переменных конденсаторов имеют очень маленькое значение (несколько десятков или сотен пФ). Значение может быть изменено на:

.
  • • Изменение площади пластин.
  • • Изменение толщины диэлектрика.

Емкость (C) ПРЯМО ПРОПОРЦИОНАЛЬНО ПЛОЩАДИ ДВУХ ПЛАСТИН , которые непосредственно перекрывают друг друга, чем больше площадь перекрытия, тем больше емкость.

Емкость ОБРАТНО ПРОПОРЦИОНАЛЬНО РАССТОЯНИЮ МЕЖДУ ПЛАСТИНАМИ. т.е. если пластины раздвигаются, емкость уменьшается.

Диэлектрик

Электроны на одной пластине конденсатора влияют на электроны на другой пластине, вызывая искажение орбит электронов внутри диэлектрического материала (изолирующего слоя между пластинами). Величина искажения зависит от природы диэлектрического материала и измеряется диэлектрической проницаемостью материала.

Разрешение

Проницаемость указывается для любого конкретного материала как ОТНОСИТЕЛЬНАЯ ДОПУСТИМОСТЬ, которая является мерой эффективности диэлектрического материала. Это число без единиц измерения, которое показывает, насколько диэлектрическая проницаемость материала больше диэлектрической проницаемости воздуха (или вакуума), для которой задана диэлектрическая проницаемость 1 (единица). Например, если диэлектрический материал, такой как слюда, имеет относительную диэлектрическую проницаемость 6, это означает, что конденсатор будет иметь диэлектрическую проницаемость, а значит, и емкость, в шесть раз больше, чем у конденсатора с такими же размерами, но диэлектриком которого является воздух.

Диэлектрическая прочность

Другой важный аспект диэлектрика — ДИЭЛЕКТРИЧЕСКАЯ ПРОЧНОСТЬ. это указывает на способность диэлектрика выдерживать напряжение, приложенное к нему, когда конденсатор заряжен. В идеале диэлектрик должен быть как можно тоньше, чтобы обеспечить максимальную емкость для данного размера компонента. Однако чем тоньше диэлектрический слой, тем легче разрушаются его изоляционные свойства. Таким образом, диэлектрическая прочность определяет максимальное рабочее напряжение конденсатора.

Максимальное рабочее напряжение (VDCwkg макс.)

При использовании конденсаторов очень важно, чтобы максимальное рабочее напряжение, указанное производителем, не превышалось. В противном случае существует большая опасность внезапного пробоя изоляции внутри конденсатора. Поскольку в это время на конденсаторе существует максимальное напряжение (отсюда и пробой), большие токи будут протекать с реальным риском возгорания или взрыва в некоторых цепях.

Заряд конденсатора.

Заряд (Q) конденсатора зависит от комбинации вышеперечисленных факторов, которые можно представить вместе как емкость (C) и приложенное напряжение (V). Для компонента данной емкости соотношение между напряжением и зарядом является постоянным. Увеличение приложенного напряжения приводит к пропорциональному увеличению заряда. Эту связь можно выразить формулой;

Q = CV

или

C = Q / V

или

V = Q / C

Где V — приложенное напряжение в вольтах.

C — емкость в Фарадах.

Q — количество заряда в кулонах.

Итак, можно найти любую из этих величин, если известны две другие. Формулы можно легко переставить, используя простой треугольник, аналогичный тому, который используется для расчета закона Ома при проведении расчетов резисторов.

Емкость

и конденсаторы с примером

Емкость и конденсаторы

Емкость — это отношение накопленного заряда к полученному потенциалу проводников.Единица измерения емкости — кулон на вольт, она называется фарадом (F).

Емкость — это скалярная величина. График, приведенный ниже, показывает соотношение полученного заряда и накопленного потенциала проводящей сферы.

Существует линейная зависимость между полученным зарядом и полученным потенциалом.Наклон графика дает нам емкость сферы.

Как я сказал ранее, фарад — это единица емкости, однако мы обычно используем (пФ) пикофарад = 10 -12 Ф, (мкФ) микрофарад = 10 -6 Ф и (нФ) нанофарад = 10 — 9 F.

Сфера радиусом r и зарядом q имеет емкость;

Конденсаторы

Конденсаторы — это устройства, предназначенные для накопления заряда.Они обычно используются в компьютерах или электронных системах. Они состоят из двух проводящих пластин, расположенных на некотором расстоянии друг от друга. Они не касаются друг друга. Когда мы соединяем отрицательно заряженную пластину с нейтральной сферой, они разделяют общий заряд до тех пор, пока потенциалы не сравняются и лепестки электроскопа не поднимутся. Затем мы размещаем пластину A на расстоянии d от B. Поскольку мы заземляем пластину, она вначале нейтральна. Поскольку B заряжен отрицательно, он воздействует на пластину A и заряжается положительно за счет индукции.Если положить между пластинами другой изолятор, например пластик, то створки электроскопа немного сомкнуты. Можно сделать вывод, что емкость пластин зависит от расстояния между пластинами.

В схеме обозначим конденсатор символом;

А батарея, которая питает разность потенциалов, обозначена символом;

Мы показываем конденсаторы и батарею в схеме, как показано ниже;


Емкость пластины зависит от;

· Площадь плит

· Расстояние между пластинами d

· Диэлектрическая проницаемость между пластинами ε º

Емкость пластин определяется по следующей формуле;


Диэлектрическая проницаемость между пластинами ε º зависит от типа материала.Например, вакуум имеет ε = 8, 85,10 -12 Ф / м, а вода имеет ε = 717,10 -12 Ф / м.

Пример: Рассчитайте емкость конденсатора, имеющего размеры 30 см X 40 см и разделенного воздушным зазором d = 8 мм.

A = 30,10 -2 м X 40,10 -2 м = 0,12 м 2

C = (8, 85,10 -12 C 2 / Н · м 2 ). 0,12 м 2 / 8,10 -3 м

С = 0, 13275.10 -9 ф

Электростатические экзамены и решения

Электрический потенциал и электрическая потенциальная энергия <Пред. Далее> Конденсаторы последовательно и параллельно
Конденсаторы коррекции коэффициента мощности

— Калькулятор размеров и формулы

Чтобы рассчитать требуемую емкость PFC, нам необходимо знать существующую реактивную мощность Q L (VAR) вашей электрической системы и выбрать желаемый коэффициент мощности.Проблема в том, что Q L не всегда известна. Есть несколько способов оценки Q L , в зависимости от того, какие другие величины известны. Мы обсудим эти методы ниже. Важно отметить тот факт, что реактивная мощность двигателя непостоянна и незначительно меняется в зависимости от нагрузки. Поэтому, чтобы избежать чрезмерной коррекции, в идеале вы должны определить значение VAR вашего двигателя на холостом ходу. К сожалению, производители редко указывают это число.

Если вы не можете получить информацию о Q L от производителя, вы можете попросить электрика измерить ток холостого хода с помощью токоизмерительных клещей и умножить результат на напряжение.Технически это будет полная ВА, но при отсутствии рабочей мощности результат будет близок к ВАР. После того, как вы определили «Q L », требуемый номинал конденсаторов PFC будет просто Qc = Q L × PF желаемый , где PF дан в виде десятичной дроби. Если вы не можете определить VAR без нагрузки, все становится немного сложнее. Вспомним из геометрии, что тангенс угла в прямоугольном треугольнике — это отношение противоположной стороны к соседней стороне. Тогда, как видно из треугольной диаграммы мощности, нескорректированные и скорректированные значения реактивной мощности задаются следующими уравнениями:

Q нескорректированный = P × tanφ 1
Q исправленный = P × tanφ 2 ,
где P — действительная мощность.Отсюда находим требуемый Qc:

Qc = Q нескорректированный -Q исправленный = P × (tanφ 1 — tanφ 2 )

У нас есть три неизвестных значения: P, φ 1 и φ 2 . Рабочую мощность P можно измерить ваттметром. Чтобы найти φ 1 , нам нужно знать гипотенузу, которая представляет полную мощность S (ВА). Итак, вам нужно измерить полный ток при полной нагрузке и умножить его на напряжение. Когда мы знаем P и S, предполагая неискаженный синусоидальный ток без гармоник, мы можем найти φ 1 = arccos (P / S) .Точно так же желаемый угол φ 2 равен φ 2 = arccos (PF 2 ), где PF 2 — целевой коэффициент мощности.

Наконец, если измерение P и VA на вашем предприятии нецелесообразно, у вас нет другого выбора, кроме как собрать значения HP, PF и эффективности из таблицы данных двигателя. Этот метод наименее точен, поскольку приведенные выше данные относятся к работе с полной нагрузкой, в то время как в действительности двигатели почти всегда недогружены. Поскольку 1 л.с. ≈ 0,746 киловатт, если вы не знаете P, вы можете оценить его как
P (кВт) = л.с. × 0.746 / η , где η — это эффективность в десятичном формате (обычно от 0,8 до 0,95). Подставляя φ1 и φ2 в наше выражение для Qc, получаем:

Qc (kVAR) = P (кВт) × [tan (arccos (PF 1 )) — tan (arccos (PF 2 ))] ,
где PF 1 и PF 2 — начальный и улучшенный PF соответственно (если у вас PF выражен в процентах, вам нужно разделить его на 100). Наш калькулятор просто реализует приведенную выше формулу. После того, как вы нашли требуемый кВАр, выберите стандартный конденсатор с равным или меньшим значением.Всегда лучше недооценить, чем перевернуть. Обратите внимание, что, хотя обычно емкость измеряется в микрофарадах, для упрощения определения размеров конденсаторов PFC производители оценивают их в киловарах (кВАр). Поскольку Ic = V / Xc и Xc = 1 / (2πFC), тогда V × I = 2πFCV 2 , где «C» в фарадах, «F» в герцах. Если мы выразим V × I в кВАр и «C» в мкФ, то соотношение между этими двумя величинами будет следующим: кВАр = 2πFC мкФ V 2 × 10 -9 .

Наш виджет предназначен только для предварительной приблизительной оценки — прочтите наш полный отказ от ответственности, указанный ниже.Вам нужен инженер, который проведет исследование, предложит решение и определит, имеет ли проект финансовый смысл.

Емкость

— Элементы схемы — Содержание MCAT

Емкость — это мера способности объекта накапливать электрический заряд. Любое тело, способное заряжаться любым способом, имеет значение емкости. Конденсаторы могут накапливать энергию при подключении батареи или источника напряжения.

Конденсатор с параллельными пластинами состоит из 2 проводящих пластин ( электродов ), разделенных изоляционным материалом ( диэлектрик ).Когда 2 электрода подключены к источнику питания, один к положительному, а другой к отрицательному, на пластинах будет накапливаться заряд. Когда равновесие достигнуто, это означает, что конденсаторы полностью заряжены.

Для конденсаторов с параллельными пластинами положительные заряды, накопленные при подключении к источнику напряжения , притягиваются к накопленным отрицательным зарядам, заряды сохраняются даже при снятии напряжения.Таким образом сохраняется энергия.

Количество электрического заряда , хранящегося в каждой из пластин, прямо пропорционально разности потенциалов между двумя пластинами (и источником напряжения):

Q =

CV
  • Q = количество заряда, хранящегося в одном конденсаторе (+ Q на одной пластине и -Q на другой пластине)
  • В = разница потенциалов между двумя пластинами
  • C = емкость конденсатора (зависит от формы конденсатора)

Конденсатор с параллельными пластинами с диэлектриком между пластинами имеет емкость, которая определяется уравнением ниже.Обратите внимание, что κ для вакуума равно 1. Диэлектрическая проницаемость для воздуха очень близка к 1, так что конденсаторы, заполненные воздухом, действуют так же, как конденсаторы с вакуумом.

Энергия, запасенная в конденсаторе, равна электрической потенциальной энергии ΔPE = qΔV. Обратите внимание, что первый заряд, помещенный на конденсатор, испытывает изменение напряжения ΔV = 0, поскольку конденсатор имеет нулевое напряжение в незаряженном состоянии. Последний заряд, помещенный на конденсатор, испытывает ΔV = V, так как конденсатор теперь имеет на нем свое полное напряжение V.Среднее напряжение на конденсаторе в процессе зарядки

Последовательные конденсаторы: Общая емкость конденсаторов, подключенных последовательно, равна сумме обратных величин каждого отдельного конденсатора.

Параллельно подключенные конденсаторы: Общая емкость конденсаторов, подключенных параллельно, равна сумме всех конденсаторов в отдельности.

Для того, чтобы конденсатор удерживал заряд, должен быть разрыв цепи между двумя его сторонами.Это прерывание может происходить в виде вакуума (отсутствие какого-либо вещества) или диэлектрика (изолятор).

Когда используется диэлектрик, материал между параллельными пластинами конденсатора поляризуется. Часть около положительного конца конденсатора будет иметь избыток отрицательного заряда, а часть около отрицательного конца конденсатора будет иметь избыток положительного заряда. Это перераспределение заряда в диэлектрике, таким образом, создает электрическое поле, противодействующее полю, создаваемому конденсатором.

Следовательно, чистое поле, создаваемое конденсатором, будет частично уменьшено диэлектриком, как и разность потенциалов на нем. С другой стороны, диэлектрик предотвращает прямой контакт пластин конденсатора (что сделало бы конденсатор бесполезным). Если он имеет высокую диэлектрическую проницаемость, он также увеличивает емкость для любого заданного напряжения.

В качестве диэлектрика можно использовать любой изолятор , но наиболее часто используемые материалы выбираются из-за их способности противостоять ионизации.Чем устойчивее материал к ионизации, тем больше он допускает работу при более высоких напряжениях. В конце концов, каждый материал имеет «точку пробоя диэлектрика», в которой разность потенциалов становится слишком большой для изоляции, и он ионизируется и пропускает ток.


Практические вопросы

Ханская академия

Конденсаторы в мониторах электрокардиографии

Нарушения ритма сердца и дефибрилляторы

Лечение электрическим полем и электропорация

MCAT Official Prep (AAMC)

Physics Question Pack Отрывок 5, вопрос 30

Physics Question Pack Отрывок 5, вопрос 33

Physics Question Pack Отрывок 9, вопрос 53

Physics Question Pack Отрывок 9 Вопрос 55

Практический экзамен 3 Раздел C / P Отрывок 2 Вопрос 5

Ключевые точки

• Конденсатор с параллельными пластинами состоит из 2 проводящих пластин (электродов), разделенных изоляционным материалом (диэлектриком).

• Конденсатор, подключенный к источнику напряжения, может накапливать энергию E = QV / 2

• Общая емкость последовательно включенных конденсаторов равна сумме обратных величин каждого конденсатора в отдельности.

• Общая емкость конденсаторов, включенных параллельно, равна сумме всех конденсаторов в отдельности.

• Диэлектрик — это изоляционный материал между электродами. С постоянным k, специфичным для каждого типа материала.


Ключевые термины

напряжение : Разность электрических потенциалов, выраженная в вольтах

электродов: проводник, через который электричество входит или выходит из объекта, вещества или области.

диэлектрик : изоляционный материал или очень плохой проводник электрического тока

электрический заряд: физическое свойство вещества, которое заставляет его испытывать силу при помещении в электромагнитное поле

разность потенциалов: разность напряжения электрического потенциала между двумя точками

электрическая потенциальная энергия: — потенциальная энергия, возникающая в результате консервативных кулоновских сил

.
Разное

Добавить комментарий

Ваш адрес email не будет опубликован. Обязательные поля помечены *